PDA

Просмотр полной версии : Центр вселенной.


Страницы : [1] 2 3

Anonymous
22.12.2000, 15:56
А кто знает, где расположен центр вселенной?И как далеко наша галактика от него расположена?

Anonymous
22.12.2000, 15:57
Что есть центр Вселенной?

Anonymous
22.12.2000, 15:58
Раз никто так и не ответил на вопрос, попробую я.

Под центром чего-либо обычно понимается точка, относительно которой некоторые свойства объекта одинаковы в любом направлении. В природе никогда не бывает полного совпадения во всех направлениях, однако, для многих объектов во Вселенной можно найти точку, относительно которой некоторые свойства тела меняются приблизительно одинаково во всех направлениях. Это и называется центром тела.

Во Вселенной наблюдения пока не обнаружили такой выделенной точки. Современная наука "видит", что Вселенная примерно одинакова во всех направлениях. Центр Вселенной пока не обнаружен. Вероятно, Вас привели к этому вопросу рассуждения о расширении Вселенной. Мы видим, что чем дальше от нас участки Вселенной, тем больше скорость их расширения. Впечатление такое, словно мы находимся в центре или неподалеку от центра расширения.

Хорошей аналогией процессов расширения Вселенной может служить такой пример. Предположим, имеется очень большой однородный лист резины, краев которого и ничего, кроме резинового листа, не видно. Начали тащить за края резины с одинаковой скоростью, в результате чего лист стал увеличиваться в размерах. Находясь в ЛЮБОЙ точке листа мы увидим, что чем дальше от нас, тем с большей скоростью удаляются от нас части листа. И это впечатление будет, как я сказал, в любой точке, то есть центра расширения не существует. Казалось бы, есть точка, относительно которой края листа расширяются одинаково. Но в том-то и дело, что в нашей модели мы не видим ничего, кроме листа и найти такую точку не можем (земной поверхности под нами просто не видим, и нет точки, относительно которой можно было бы заметить движения листа).

Аналогично и со Вселенной. Если и существует точка, относительно которой "края" Вселенной расширяются одинаково, то из наблюдений мы пока не можем обнаружить этой точки, т.к. видим изнутри лишь расширяющуюся Вселенную и ничего более (ни краев Вселенной, ни реперной точки вне Вселенной). Поэтому, находясь ВНУТРИ равномерно расширяющейся Вселенной и не видя ничего, кроме "ближайших" расширяющихся частей Вселенной, мы не можем ничего сказать, существует ли то, что Вы назвали центром Вселенной.

Anonymous
22.12.2000, 15:59
Определение центра как точки,относительно которой "некоторые" свойства объекта одинаковы,не совсем верно.
Допустим,если мы из центра кристалла будем двигаться в разных направлениях,то увидим,что свойства кристалла будут изменяться неодинаково.
Как известно,это явление назывется анизотропией.
Или возьмём,например,сферу,
из которой откачан воздух,и примем,что пространство внутри неё изотропно.Таким образом под указанное определение центра подпадает вообще любая точка сферы,так как относительно любой точки все свойства одинаковы.
Теперь по-поводу аналогии с листом резины.
Данное сравнение применимо только тогда,когда не обнаружены границы объекта.Однако в случае со Вселенной такие границы имеются,равно как имеется диаметр,радиус(а именно радиус К из теории Фримана)и др.
Теперь,что касается самой проблемы.В своё время Эйнштейн и другие отрицали наличие центра,ссылаясь на однородность и изотропность Вселенной.Подобные действия,думается,надо считать некой абстракцией или приближением что-ли для возможности обоснования и расчёта(как в тероии вероятности).Но,строго говоря,центр всё-таки должен быть,хотя бы с геометрической точки зрения:во-первых,как уже указывалось границы у Вселенной имеются(пусть они даже не соответствуют современным цифрам,но хотя бы с чисто логической точки зрения они должны быть,т.к.Вселенная образовалась путём расширения из конечного объёма(10^-35 что-ли(правда непонятно,как можно говорить о конечном объёме если за пределами Вселенной не было пространства(ну,да неважно))).Проблема,видно в том,что центр просто ещё не нашли(а если и не ищут,то это совершенно возмутительно).Поэтому,товарищи,рекомендую срочно засесть за телескопы,пока нас Американцы не обогнали.

Anonymous
22.12.2000, 16:00
Балбалом Турмаляк!

Если кристалл обладает центральной симметрией, то расстояние от центря кристалла до границ кписталла и будет тем "некоторым" свойством. Анизотропия относится к ФИЗИЧЕСКИМ свойствам кристалла и к ГЕОМЕТРИЧЕСКОМУ центру отношения не имеет!

Сфера имеет центр лишь в геометрической точке, относительно которой расстояния до сферы одинаковые. Расстояние и является "свойством", по которому определяется центр. Имеется там воздух, или он откачан - неважно!

Кто и когда из НАБЛЮДЕНИЙ обнаружил край Вселенной. Дайте, пожалуйста ссылку.

Anonymous
22.12.2000, 16:00
В догонку к предыдущему.
А как вообще из наблюдений определить, что мы достигли края Вселенной? Ведь от далеких областей мы получаем лишь электромагнитное излучение! Как смогли определить, что края есть?

Anonymous
22.12.2000, 16:01
Ну,во-первых у вас там не было указано,что под "некоторыми" свойствами подразумевается расстояние.Вот я грешным делом и подумал,что имеются ввиду физические свойства(говоря откровенно,до сих пор я вообще не считал расстояние свойством).
Во-вторых.Вот вы пишете":если кристалл обладает центральной симметрией...",а если не обладает,как,скажем,турмалин?Если вообще имеется кривая фигура,не имеющая симметрии?Тогда расстояние от краёв до центра будет совсем не постоянным.Или может быть центра вовсе не будет?
Что касается краёв.Конркетно указать источник не могу,так как читал это в книгах,которых теперь у меня нет.Наиболее прямо о краях говорилось в "Мире животных"И.Акимушкина.Не помню в каком томе.Там говорилось,что современная астрономия обладает возможностями,позволяющими заглянуть гораздо дальше вглубь Вселенной,чем находятся самые далёкие объекты(!).Однако,поскольку дальше ничего не видно,то в книге делался вывод,что дальше,собственно,ничего и нет,и что эти объекты и являются краями.На то время самым далёким был квазар ОН-471(92 год).В учебнике астрономии(кажется 96 года)приводится другой квазар-3С-273(или как-то так).
Так.Тут меня отвлекают...

Anonymous
22.12.2000, 16:01
А мне собственно нужно было только знать, обнаружили этот центр или нет.
Советую почитать теорию относительности.Там все подробно описывается на счет пространства-времени.

Anonymous
22.12.2000, 16:02
Дромедар Блубакеш-133!! Он же ... Вас что, преследуют? Кликуху меняете часто!

1. Забавно! "Если ничего не видно, то там ничего и нет". Мне друг с Украины не пишет недели две. Значит Украины уже нет! Отсутствие сведений означает лишь одно: сведения отсутствуют. И только! Отсутствие объекта так не установить! Подобное "наблюдение" края Вселенной просто абсурд!
2. В каждом объекте есть некоторое количество "центров": центр масс, центр инерции, центр давления набегающего потока ... Указанные центры есть у любого твердого тела, даже у асимметричного кристалла. Признаки определения центра могут быть практически любыми.

Anonymous
22.12.2000, 16:02
По моим расчотам мы находимся достаточно близко от центра вселенной,так как скорость движения нашей голактики слишком мала и
все голактики нашей местной группы движутся примеро одинаково со скоростью 600000км/ч.Я думаю вы знаете что чем дальше от нас находится объект,тем быстрее он от нас уддоляется и тем больше его красное смищение в спектре.Если будут интересные мопросы
или расказы пишите мне на mail.

Anonymous
22.12.2000, 16:03
Я недавно читал книгу не помню названия,но помню лиш то что наша вселенная существует уже
25млрд.лет и её возрост с развитеем науки тоже постоянно ростёт.Самым далёким объектом евляются протогалактикика растояние до неё 17млрд.световых лет,её красное смещение равно Z=6.А вот самый далёкий квазаром евляется PC1247+34 его красное смещение Z=4.9 а растояние до него 15млрд.световых лет.Но эти данные были ещё в 1998году,за два года я думаю нашли ещё более далёкие объекты.Что касается квазара 3с273 то это ближайший квазар и звестный учонным до него 3млрд.световых лет.

megabobr
10.11.2009, 20:32
а как вы думаете что было до большого взрыва если он вообще был ?

megabobr
10.11.2009, 20:50
Я недавно читал книгу не помню названия,но помню лиш то что наша вселенная существует уже
25млрд.лет и её возрост с развитеем науки тоже постоянно ростёт.Самым далёким объектом евляются протогалактикика растояние до неё 17млрд.световых лет,её красное смещение равно Z=6.А вот самый далёкий квазаром евляется PC1247+34 его красное смещение Z=4.9 а растояние до него 15млрд.световых лет.Но эти данные были ещё в 1998году,за два года я думаю нашли ещё более далёкие объекты.Что касается квазара 3с273 то это ближайший квазар и звестный учонным до него 3млрд.световых лет.

А международная группа исследователей обнаружила самый удалённый квазар из известных на сегодняшний день. Расстояние до него столь велико, что свету требуется 9/10 времени жизни Вселенной для того чтобы дойти до земли. Они надеются , что этот квазар ответит на вопросы , касающиеся загадочного периода в развитии Вселенной , называемого Эпоха Реионизации.;)

megabobr
10.11.2009, 21:10
А мне собственно нужно было только знать, обнаружили этот центр или нет.
Советую почитать теорию относительности.Там все подробно описывается на счет пространства-времени.

в общей теории относительности внешнего пространства - времени вообще нет: оно само является динамической переменной теории , зависящий от характеристик находящимся в нём квантовых систем.O:)

Ушедший
10.11.2009, 22:09
Разговариваете с призраками :) Посмотрите на даты постов.

Юрий
11.11.2009, 00:06
Всего девять лет, ерунда то какая :).

thirtyseconds
11.11.2009, 00:08
Дык луч света наверно дошел. Товарищ видимо с Сириуса :)

megabobr
11.11.2009, 11:15
Что за пределами вселеной ? наверное там ничего нет ни времени ни матении совершено ничего. А может я и ошибаюсь. А не могу понять вечность , безграничност и пустоту где нет ничего.

monstr
11.11.2009, 11:17
Фиг с ним, что там за краем. А вот вопрос - где именно центр? Галактики разбегаются. Куда направлены выкторы? Чтение теории меня только запутало. Кто тут разбирается во вселенных? :), помогайте.

megabobr
11.11.2009, 11:35
Фиг с ним, что там за краем. А вот вопрос - где именно центр? Галактики разбегаются. Куда направлены выкторы? Чтение теории меня только запутало. Кто тут разбирается во вселенных? :), помогайте.

Фик с центром может его и нет , а что было до появления вселеной#-o :sad:

ZamaZzZka
11.11.2009, 11:36
Нету центра, он везде :)
Представьте сферу, все галактики лежат на ней. Сфера раздувается, галактики друг от друга отдаляются.

thirtyseconds
11.11.2009, 11:36
Существование таких векторов означало бы наличие какой-то выделенной точки, центра вселенной (взрыва). Такой точки нет, нет и векторов. Каждая точка пространства удаляется от других.

http://cosmologiya.narod.ru/nocenter.html
http://cosmologiya.narod.ru/cosmolog.htm - Учебник космологии Неда Райта.

megabobr
11.11.2009, 11:41
у меня племяница сросила откуда появилась вселеная , а я пытался ответить , в общем у меня не вышло и меня совесть мучает

monstr
11.11.2009, 11:41
Нету центра, он везде :)
Представьте сферу, все галактики лежат на ней. Сфера раздувается, галактики друг от друга отдаляются.
Представил. У шара есть центр. Оттуда всё и удаляется. Если мы в центре - от нас всё удаляется. Если мы с краю, то одно удаляется быстро (другой край), другое - медленно.

Пока не понял, как может не быть центра расширения, если оперировать обычными понятиями (три измерения, время и т.п.), так сказать "обывательскими".

ZamaZzZka
11.11.2009, 11:44
У шара да, у вселенной центра нет, она находится на шаре.

monstr
11.11.2009, 11:50
У шара да, у вселенной центра нет, она находится на шаре.
Не есть понимать опять...
Опять таки, если предположить, что нет Эйнштейна, его ОТО и прочего, то каким образом, находясь мы хоть внутри, хоть снаружи раздуваемого шара, мы не видим векторов разбеганий вещества (звёзд, галактик и т.п.)?

Скажем, звезда Бернарда. Бежит себе направо (или налево, не суть, ведь всё относительно) со скоростью около N километров в секунду (относительно Солнца). Но при этом, кроме её собственного вектора движения, она так же удаляется от Солнца ... от Сириуса, от всего! Вселенная "раздувается", но центр-то в данном случае не Солнце, раздувается она откуда? :)

Сорри, но и правда не могу понять, есть ли в рамках классической механики (которую я понимаю) объяснение, как может не быть центра разбегания?

Ar-Gen-Tum
11.11.2009, 11:50
Нету центра, он везде :)
Представьте сферу, все галактики лежат на ней. Сфера раздувается, галактики друг от друга отдаляются.
Все ждал кто первым выдаст такой ответ. :)
Ждать долго не пришлось. Плоды единообразного просвещения. :)
Т.е. вроде-бы мир трехмерный, а когда доходит до объяснений
расширения Вселенной, то "Представьте сферу, все галактики лежат на ней."(c)

thirtyseconds
11.11.2009, 11:54
Скажем, звезда Бернарда. Бежит себе направо (или налево, не суть, ведь всё относительно) со скоростью около N километров в секунду (относительно Солнца). Но при этом, кроме её собственного вектора движения, она так же удаляется от Солнца ... от Сириуса, от всего! Вселенная "раздувается", но центр-то в данном случае не Солнце, раздувается она откуда? :)

Вообще расширение вселенной начинает проявляться только в масштабах галактических скоплений. Говорить о удалении звезд друг от друга и даже галактик в пределах Местного скопления бесмысленно.


Сорри, но и правда не могу понять, есть ли в рамках классической механики (которую я понимаю) объяснение, как может не быть центра разбегания?

Самый популярный научно-популярный пример ( :) )Замазка уже привела. Возьмите сдутый шарик. Наставьте на нем точки (галактики) и начните надувать. Каждая галактика будет удаляться от остальных. Причем чем дальше они будут друг от друга тем больше будет скорость удаления. выделенного центра не будет.

monstr
11.11.2009, 11:55
Все ждал кто первым выдаст такой ответ. :)
Ждать долго не пришлось. Плоды единообразного просвещения. :)
Т.е. вроде-бы мир трехмерный, а когда доходит до объяснений
расширения Вселенной, то "Представьте сферу, все галактики лежат на ней."(c)
Во-во.
Я нисколько не против ОТО и прочего. Я даже не против магии и Гарри Поттера. Я просто хочу узнать, можно ли объяснить расширение вселенной именно трёхмерной, классической моделью?

ZamaZzZka
11.11.2009, 11:55
Ну такие объекты, как звезды в галактиках, галактики, типа Андромеды и нашей, т.е. связанные гравитационно, не удаляются, а даже наоборот приближаются. А не связанные удаляются друг от друга.
Ну вот, самое простое - это представить галактики на сфере. Каждая точка на этой сфере будет центром, от которого с одинаковым ускорением удаляются остальные галактики. Куда не тыркни, везде центр.

ZamaZzZka
11.11.2009, 11:56
Все ждал кто первым выдаст такой ответ. :)
Ждать долго не пришлось. Плоды единообразного просвещения. :)
Т.е. вроде-бы мир трехмерный, а когда доходит до объяснений
расширения Вселенной, то "Представьте сферу, все галактики лежат на ней."(c)
Мир от этого не перестает быть трехмерным :)

monstr
11.11.2009, 11:57
Вообще расширение вселенной начинает проявляться только в масштабах галактических скоплений. Говорить о удалении звезд друг от друга и даже галактик в пределах Местного скопления бесмысленно.


Странное заявление. Сила разбегания или есть или нет. Как она может действовать на галактики и не действовать на одуванчик на планете Земля? Она квантовая /атомарная чтоли? :)


Самый популярный научно-популярный пример ( :) )Замазка уже привела. Возьмите сдутый шарик. Наставьте на нем точки (галактики) и начните надувать. Каждая галактика будет удаляться от остальных. Причем чем дальше они будут друг от друга тем больше будет скорость удаления. выделенного центра не будет.
Пример не понял, выше объяснил.
Мы, к примеру в одной из этих галактик. Смотрим на противоположную - она несётся на двойной скорости от нас.

ZamaZzZka
11.11.2009, 11:58
Сила притяжения в скоплениях галактик сильнее.

monstr
11.11.2009, 11:59
Сила притяжения в скоплениях галактик сильнее.
Э... есть параллельная тема, там как раз обсуждают, есть ли границы гравитации. Моё мнение, что их нет, что гравитационное взаимодействие есть между тем самым одуванчиком на Земле и мега-галактическим одуванчиком на третьей планете 1002023 звезды в Андромеде :)

Ну такие объекты, как звезды в галактиках, галактики, типа Андромеды и нашей, т.е. связанные гравитационно, не удаляются, а даже наоборот приближаются. А не связанные удаляются друг от друга.
Ну вот, самое простое - это представить галактики на сфере. Каждая точка на этой сфере будет центром, от которого с одинаковым ускорением удаляются остальные галактики. Куда не тыркни, везде центр.

Шарик есть под рукой? :) Расширение в точке рядом с наблюдателем будет медленнее, чем в точке на противоположной от наблюдателя стороне.

ZamaZzZka
11.11.2009, 12:07
Э... есть параллельная тема, там как раз обсуждают, есть ли границы гравитации. Моё мнение, что их нет, что гравитационное взаимодействие есть между тем самым одуванчиком на Земле и мега-галактическим одуванчиком на третьей планете 1002023 звезды в Андромеде :)
Не поняла :) Есть сила, удерживающая галактики в скоплениях, звезды в скоплениях, вместе. Она вскоплениях больше силы отталкивайния. Не в скоплениях - меньше :)



Шарик есть под рукой? :) Расширение в точке рядом с наблюдателем будет медленнее, чем в точке на противоположной от наблюдателя стороне.
Ну, все верно. А поставь наблюдателя в любую другую точку на этом шарике, будет там же все то же самое. Тот же центр, те же законы.

Ar-Gen-Tum
11.11.2009, 12:09
Мир от этого не перестает быть трехмерным :)
Видимо, да. :) Скажем так количество измерений не менее 3-х.
Значит гипотеза о БВ выглядит примерно так:
Вначале Вселенная находилась в точке, где наши законы физики не действуют.
Типа в состоянии сингулярности.
Потом Вселенная стала раздуваться. Обратите внимание - раздуваться
во все стороны(как минимум 3-и независимых, взаимноперпендикулярных направления).
Это вполне может означать, что та точка от которой происходит расширение
и есть центр.
А вобще-то теория(именно теория) БВ - это терия-"заглушка"
в противовес "божественному сотворению мира".

ZamaZzZka
11.11.2009, 12:11
http://astronet.ru/db/msg/eid/FK86/cosmology
п.2 Нестационарность Вселенной.

http://astronet.ru/db/msg/1166311

ZamaZzZka
11.11.2009, 12:15
Это вполне может означать, что та точка от которой происходит расширение
и есть центр.
А вобще-то теория(именно теория) БВ - это терия-"заглушка"
в противовес "божественному сотворению мира".
Ну может она сразу расширялась как поверхность сферы :)

Ну теория теорией, она основана на наблюдаемых фактах. Они интрпретируются как расширение Вселенной. Можете предложить что-то другое, основываясь на этих фактах? Чтобы все предусмотреть, чтобы ничто ничему не противоречило? Пока что с этим справляется только эта теория.

thirtyseconds
11.11.2009, 12:16
Пример не понял, выше объяснил.
Мы, к примеру в одной из этих галактик. Смотрим на противоположную - она несётся на двойной скорости от нас.

Ну автомобиль тоже на меня несется. Никто ж не удивляется почему расширение вселенной его не тащит назад. Дело только в масштабах. Есть масштабы на которых закон хаббла начинает доминировать.

EDIT : Неверно прочитал ваше сообщение.

Противоположная галактика (туманность Андромеды, если вы ее имели ввиду) несется как раз таки _к_ нам. Все галактики в местном скоплении имеют скорости, которые обусловлены перемещением относительно центра масс системы. То есть если вы попытаетесь доказать скоростями местных галактик Закон Хаббла, то ничего не выйдет. Для того что он начал доминировать нужно иметь дело с удаленными от нашей системы галактиками и их скоплениями.
Можно не согласиться с интерпретацией этого явления, но в целом она представляет строгий наблюдательный факт - с определенного расстояния вы не найдете НИ ОДНОЙ галактики, которая бы приближалась к нам.

igor_da_bari
11.11.2009, 12:47
Сорри, но и правда не могу понять, есть ли в рамках классической механики (которую я понимаю) объяснение, как может не быть центра разбегания?

При личной встрече с листком бумаги и ручкой берусь объяснить. Но мне нада не меньше литра пива или 200 грамм коньяку :D

ZamaZzZka
11.11.2009, 12:49
Меня возьмите, мне тоже интересно послушать :)

igor_da_bari
11.11.2009, 12:51
Меня возьмите, мне тоже интересно послушать :)

Тада да. Тада вам проще будет: каждый по поллитре в меня вольёт - и дело в шляпе :D

thirtyseconds
11.11.2009, 12:51
Нехорошие вы люди... Пива с каньяком трескаете. А я тут в одиночку давиться должен.

monstr
11.11.2009, 12:51
А мне - попить :)

Ок, замётано. Как объективы буду отдавать.

megabobr
11.11.2009, 12:53
[QUOTE=Anonymous;35701]Что есть центр Вселенной?[/QUO


Лично я считаю центра нет , вселеная это нечто безпредельное там не существует таких понятий которые существуют для нас .O:)

monstr
11.11.2009, 12:55
Лично я считаю центра нет , вселеная это нечто безпредельное там не существует таких понятий которые существуют для нас .O:)
"Там" - это где? Мы не в этой Вселенной? :)

M.Sergey
11.11.2009, 13:05
Я слышал мнение, что любую точку вселенной можно считать центром.

megabobr
11.11.2009, 13:56
"Там" - это где? Мы не в этой Вселенной? :)
На этот вопрос двумя фразами не ответить . Советую прочесть труды А.Клизовского Основы Миропонимания новой эпохи , либо Н.Рериха Семь Великих тайн космоса. Глава Дни и Ночи Брахмы.O:)

monstr
11.11.2009, 13:59
Ойоо... Удаляюсь из темы :)

Сразу вспомнились слова Джея из кино "Догма": "Вокруг столько аморальных девиц, а мы кликушу подцепили" :) :)
Я думал мы науку обсуждаем, а не ....

Ar-Gen-Tum
11.11.2009, 14:14
Ну может она сразу расширялась как поверхность сферы :)

Ну теория теорией, она основана на наблюдаемых фактах. Они интрпретируются как расширение Вселенной. Можете предложить что-то другое, основываясь на этих фактах? Чтобы все предусмотреть, чтобы ничто ничему не противоречило? Пока что с этим справляется только эта теория.
Натали & All. Допустим есть наблюдаемый факт, что удаленные скопления галактик удаляются от нас.
Допустим - потому что это так трактуется Хаббловское покраснение спектра.
Существует также принципиальная возможность наблюдать вселенную
практически во всех направлениях. Кроме того сектора, который перекрыт Млечным Путем.
Таким образом есть принципиальная возможность составить 3-х мерную
карту видимой части вселенной. Для каждого объекта как-бы известны
расстояние и скорость убегания.
Из этого находим центр разбегания. :)
Теперь по поводу понятия центр:
Понятие центр применимо лишь к окружности на плоскости и к сфере в 3-х мерном пространстве.
Это понятие также распространяется на круг, ограниченный окружностью
и на шар, ограниченный сферой.
А так-же на их частные элементы, типа дуги окружности или сегмента сферы.
Посему объяснение расширения, подменяющее объемную фигуру
поверхностью принципиально не имеющей центра, является некорректным. Хотя и логичным
образом устраняет понятие центра. :)
Вот Земля геоид. У нее есть центр?
Вобще центр Земли мог-бы находится в точке пересечения множества продолжений линий отвеса из разных точек.
Но эти продолжения в одной точке не пересекуться. Ибо Земля геоид.
Но вокруг всех точек пересечения можно описать сферу
и центр этой сферы мог-бы в принципе служить центром некоторой
усредненной сферы, аппроксимирующей земную поверхность
с некоторой точностью.
Так-же и для Вселенной(или вернее для Метагалактики) можно приблизительно
указать некий центр. Если ее масса и протяженность конечны.

monstr
11.11.2009, 14:21
Допустим - потому что это так трактуется Хаббловское покраснение спектра.


Согласен. И думал, так и сделали давно. Честно сказать, для меня стало открытием (буквально позавчера), что центра нет и все жмут плечами и говорят "ну да, так и должно быть".


Для каждого объекта как-бы известны
расстояние и скорость убегания.
Из этого находим центр разбегания. :)


Я слабо знаком с этой теорией, но мне казалось, что мы знаем смещение = знаем скорость. А на практике принято считать, чем больше смещение, тем дальше объект. То есть, карту мы составить не сможем, ибо у нас только скорость (или только расстояние). Опять таки, поправьте меня, где ошибся, не уверен в сказанном.

thirtyseconds
11.11.2009, 14:30
Аналогия с поверхностью сферы это просто аналогия. Она не отражает характеристики того предмета, для объяснения которого мы ее привлекли. Это просто удобный способ показать как это любая точка может удаляться от любой другой, да еще и с ускорением.
Подробно по геометрии вселенной в любом учебнике должно быть, я еще тоже особо не разбирался. Знаю только что существуют разные модели, среди которых какой-то общепринятой не существует.

Если кому-то очень хочется иметь центр взрыва, то дело то здесь как раз за малым - найти векторы, определить центр. Нужные для этого данные имеются. Результат предсказуем - в центре "взрыва" (вселенной) будет Земля :) Которую видимо снова придется поставить на черепаху.

mvp
11.11.2009, 14:37
Все векторы указывают на то, что мы в центре Вселенной! В данном случае, центр находится на Таганке и иногда перемещается под Домодедово. :D

igor_da_bari
11.11.2009, 14:41
Согласен. И думал, так и сделали давно. Честно сказать, для меня стало открытием (буквально позавчера), что центра нет и все жмут плечами и говорят "ну да, так и должно быть".

Это, конечно, сделали давно и убедились, что найденный ТАКИМ ОБРАЗОМ центр находится в районе Солнечной системы.

Что отнюдь не означает, что РЕАЛЬНЫЙ ЦЕНТР расширения находится именно в Москве.

Поскольку если бы любой наблюдатель, находясь в сколь угодно далеко удаленной от нас точке Вселенной, тоже построил бы вектора скорости и нашел точку их пересечения, он обнаружил бы, что центр расширения находится точно у него на переносице :)

Я слабо знаком с этой теорией, но мне казалось, что мы знаем смещение = знаем скорость. А на практике принято считать, чем больше смещение, тем дальше объект. То есть, карту мы составить не сможем, ибо у нас только скорость (или только расстояние). Опять таки, поправьте меня, где ошибся, не уверен в сказанном.

Для более или менее близких галактик мы знаем и скорось, и расстояние. Для совсем удаленных объектов, скажем, какого-нибудь квазара, до которого расстояние 10 миллиардов СЛ, расстояние вычисляется по красному смещению (и даже измеряется в нем). Вычисления базируются на законе, полученном по ЭКСПЕРИМЕНТАЛЬНОЙ зависимости скорости (или, что то же самое, смещения) от расстояния.

mvp
11.11.2009, 14:54
Вот еще бы поточнее установили эту зависимость, а то разброс большой от 50 - 100км/c на МПс. Где-то читал, что многие предпочитают считать центром нашей метагалактики скопление в Деве Abell 2218 (это там где М87.)

monstr
11.11.2009, 14:55
Для более или менее близких галактик мы знаем и скорось, и расстояние. Для совсем удаленных объектов, скажем, какого-нибудь квазара, до которого расстояние 10 миллиардов СЛ, расстояние вычисляется по красному смещению (и даже измеряется в нем). Вычисления базируются на законе, полученном по ЭКСПЕРИМЕНТАЛЬНОЙ зависимости скорости (или, что то же самое, смещения) от расстояния.
Для более-менее близких объектов и вычисляется не по КС, как я понимаю? Методов вычисления расстояния масса, ты их приводил "на страницах старлаба", в википедии есть на то статья, а то и не одна. Стоит начать, например, с этой (http://ru.wikipedia.org/wiki/%D0%A8%D0%BA%D0%B0%D0%BB%D0%B0_%D1%80%D0%B0%D1%81% D1%81%D1%82%D0%BE%D1%8F%D0%BD%D0%B8%D0%B9_%D0%B2_% D0%B0%D1%81%D1%82%D1%80%D0%BE%D0%BD%D0%BE%D0%BC%D0 %B8%D0%B8#.D0.9C.D0.B5.D1.82.D0.BE.D0.B4_.D0.BE.D0 .BF.D1.80.D0.B5.D0.B4.D0.B5.D0.BB.D0.B5.D0.BD.D0.B 8.D1.8F_.D1.80.D0.B0.D1.81.D1.81.D1.82.D0.BE.D1.8F .D0.BD.D0.B8.D1.8F_.D0.BF.D0.BE_.D0.B3.D1.80.D0.B0 .D0.B2.D0.B8.D1.82.D0.B0.D1.86.D0.B8.D0.BE.D0.BD.D 0.BD.D1.8B.D0.BC_.D0.BB.D0.B8.D0.BD.D0.B7.D0.B0.D0 .BC). Не суть. Суть именно в том, что мы приняли, что КС есть расстояние, в то время как это скорость убегания объекта от нас. То есть карту векторов скоростей мы составить не можем, центр найти не можем.

Я верно понял?

Ar-Gen-Tum
11.11.2009, 15:10
Это, конечно, сделали давно и убедились, что найденный ТАКИМ ОБРАЗОМ центр находится в районе Солнечной системы.

Что отнюдь не означает, что РЕАЛЬНЫЙ ЦЕНТР расширения находится именно в Москве.

Поскольку если бы любой наблюдатель, находясь в сколь угодно далеко удаленной от нас точке Вселенной, тоже построил бы вектора скорости и нашел точку их пересечения, он обнаружил бы, что центр расширения находится точно у него на переносице :)
...

И что сие означает? :)
Если множество наблюдателей, находящихся внутри ограниченного по массе
и протяженности пространства,
указывают на то, что именно они находятся в центре, значит только то
что их понятия об этом пространстве не верны.
Например наблюдатели из Москвы, Санкт-Петербурга и Нью-Йорка
примерно одинаково определят направление на центр Земли.
А совместными усилиями и определят его примерное положение.
Т.е. центр Земли независим от наблюдателя.

igor_da_bari
11.11.2009, 15:19
Для более-менее близких объектов и вычисляется не по КС, как я понимаю?

Нет, конечно. Методов, действительно, вагон и маленькая тележка. По цефеидам, например. У которых светимость жестко связано с периодом изменения блеска. Меряешь период -> определяешь светимость -> измеряешь видимый блеск -> определяешь расстояние. А цефеиды и в очень далеких галактиках видны.

Красное смещение тут совершенно ни при чем. По красному смещению определяют скорость. В предположении, что оно целиком вызвано эффектом Допплера. То есть КС и скорость - это по сути дело одно и то жже.

Знаешь расстояние, знаешь скорость для многих тыщ объектов. Для каждого из них откладываешь точку в системе координат, в которой абсцисса - это расстояние, а ордината - скорость (КС). Получаешь зависимость скорости (КС) от расстояния. И видишь, что это - более или менее прямая линия, проходящая через начало координат. Ее наклон - это и есть постоянная Хаббла. Сейчас она известна с бОльшей точностью, чем указано уважаемым mvp выше. 50-100 - это такая неопределенность была, сколько я помню, в начале 90-х. Сейчас точно не знаю, врать не буду, давно не слежу, но когда я последний следил, было что-то типа 70 плюс-минус 10. Или очень близко. В тырнете легко найти, я думаю.

Если объект совсем далек, то расстояние до него трудно независимо определить. Тогда измеряется КС (скорость) и, зная зависимость ее от расстояния, находишь расстояние. Но это, повторяю, только для очень удаленных объектов.

Суть именно в том, что мы приняли, что КС есть расстояние, в то время как это скорость убегания объекта от нас. То есть карту векторов скоростей мы составить не можем, центр найти не можем.

Я верно понял?

Нет. Не верно. См. выше. Мы не принимали, что КС есть расстояние. Мы приняли, что КС есть скорость. Независимо измеряется расстояние и КС. Поэтому карту векторов составить - раз плюнуть (тангенциальную составляющую скорости можно померять прямо и непосредственно). Что и делалось много-много раз.

igor_da_bari
11.11.2009, 15:24
И что сие означает?

Абсолютно ничего :) Просто это так - и все.

Кстати, угадайте с трех раз, в чем принципиальная разница между группой наблюдателей из NY, Москвы и Питера и группой наблюдателей, расположенных в трех галактиках, расстояние между которыми, ну скажем, порядка 300 МПс?

monstr
11.11.2009, 15:28
Поэтому карту векторов составить - раз плюнуть (тангенциальную составляющую скорости можно померять прямо и непосредственно). Что и делалось много-много раз.
... и эти исследования показали... :) Не томи, Игорь. Ты же знаешь, а я нет. Страданиям моим не будет конца, если не скажешь, где центр вселенной :)

Шучу. А по делу. Исследования показали, что центра нет и всё движется броуновски или же есть место, где произошёл БВ?

igor_da_bari
11.11.2009, 15:32
... и эти исследования показали... :) Не томи, Игорь. Ты же знаешь, а я нет. Страданиям моим не будет конца, если не скажешь, где центр вселенной :)

Шучу. А по делу. Исследования показали, что центра нет и всё движется броуновски или же есть место, где произошёл БВ?

Олег, я ж тебе синим по желтому сказал (и не только я), что в результате центр расширения оказывается в районе Солнечной системы. Но это не значит, что пи.....нуло именно тут у нас :D

monstr
11.11.2009, 15:36
Олег, я ж тебе синим по желтому сказал (и не только я), что в результате центр расширения оказывается в районе Солнечной системы. Но это не значит, что пи.....нуло именно тут у нас :D
Я так и думал, что мы уникальные перцы!

В детстве вообще считал (как и многие дети, кстати), что мир создан исключительно для меня. Что всё вокруг происходящее есть спектакль, который для меня играют.

Ну, понятное дело, что сейчас этот спектакль играют не мне :(

mvp
11.11.2009, 15:38
По цефеидам, например. У которых светимость жестко связано с периодом изменения блеска. Меряешь период -> определяешь светимость -> измеряешь видимый блеск -> определяешь расстояние. А цефеиды и в очень далеких галактиках видны.
Тут еще одна проблема, надо учитывать межзвездное поглощение пылью и вносить поправку.

igor_da_bari
11.11.2009, 15:50
Я так и думал, что мы уникальные перцы!

В детстве вообще считал (как и многие дети, кстати), что мир создан исключительно для меня. Что всё вокруг происходящее есть спектакль, который для меня играют.

Ну, понятное дело, что сейчас этот спектакль играют не мне :(

Что перцы мы уникальные - с этим спорить трудно. Вот взять хоть меня... :D

Но расширение Вселенной тут ни при чем. Поскольку, как я уже говорил, любой наблюдатель в любой точке Вселенной получит тот же результат: "центр", из которого "выходят" все векторы, окажется в точности на его огороде...

monstr
11.11.2009, 15:51
Что перцы мы уникальные - с этим спорить трудно. Вот взять хоть меня... :D

Но расширение Вселенной тут ни при чем. Поскольку, как я уже говорил, любой наблюдатель в любой точке Вселенной получит тот же результат: "центр", из которого "выходят" все векторы, окажется в точности на его огороде...
Ты забыл добавить "наверное" или "теоретически", или ещё как-нибудь обозвать, что кроме как из нашей точки мы, пока что, не наблюдали из других. Или я что-то пропустил? :) :)

igor_da_bari
11.11.2009, 15:54
Тут еще одна проблема, надо учитывать межзвездное поглощение пылью и вносить поправку.

Это естественно. Там не одна проблема. Их много, как всегда. Я только про принцип говорил. Эт всегда так - принцип, как правило, простой, но практически его реализовать неизмеримо сложнее, потому что появляется 1001 практический ньюанс :)

Ar-Gen-Tum
11.11.2009, 15:59
...
Кстати, угадайте с трех раз, в чем принципиальная разница между группой наблюдателей из NY, Москвы и Питера и группой наблюдателей, расположенных в трех галактиках, расстояние между которыми, ну скажем, порядка 300 МПс?
А чего гадать-то.
Единственная принципиальная разница в том,
что группы наблюдателей из галактик не могут за разумное время
сличить свои измерения между собой.
Т.е. пока сличают все изменится.
И еще то, что нарушается одновременность измерений. Опять-же сложно
договорится об одновременности измерений.
Вобщем как-то так.

Это естественно. Там не одна проблема. Их много, как всегда. Я только про принцип говорил. Эт всегда так - принцип, как правило, простой, но практически его реализовать неизмеримо сложнее, потому что появляется 1001 практический ньюанс :)
Вот, вот, вот.
Плюс и теоретически не все однозначно и очевидно.
Посему гипотезы надо называть гипотезами. А то получается, что величайшим умам не понятно,
а студент/школьник говорит - "В учебнике все написано. Ведь не дураки-же пишут." . :)

igor_da_bari
11.11.2009, 16:04
Ты забыл добавить "наверное" или "теоретически", или ещё как-нибудь обозвать, что кроме как из нашей точки мы, пока что, не наблюдали из других. Или я что-то пропустил? :) :)

Строго говоря, из других точек мы не наблюдали и вряд ли когда отнаблюдаем. Однако, если все так, как мы думаем, то и это тоже так. Поскольку этот эффект не сам по себе. Он сязан с другими, вполне наблюдаемыми. Поэтому думаем мы именно так не просто так - у нас есть довольно веские основания.

Собственно, этот эффект ты и сам легко можешь "пощупать", если у тебя хорошее пространственное воображение (у меня вот оно нулевое). Нарисуй все на бумажке. И добавь время. Учти тот факт, что время тут у тебя и на удаленном объекте различается. И если кто-то проводит эксперимент по построению векторов там, то это будут другие вектора. В масштабе Вселенной мы не можем пренебречь тем фактом, что объекты, для которых мы наши вектора строим, находятся В РАЗНОМ ВРЕМЕНИ. M31, к примеру, мы видим такой, какой она была 2000000 лет назад и другой УВИДЕТЬ В ПРИНЦИПЕ НЕ МОЖЕМ. И скорость ее мы меряем такой. какой она была 2000000 лет назад. Вот если это взять в расчет и осмыслить, то не покажется удивительным тот факт, что любой наблюдатель в любой точке получит центр у себя в холодильнике :)

monstr
11.11.2009, 16:08
А вот про это я не подумал. Поразмыслю.
Ты прав, Андромеды уже давно может не быть вообще :)

igor_da_bari
11.11.2009, 16:17
А чего гадать-то.
Единственная принципиальная разница в том,
что группы наблюдателей из галактик не могут за разумное время
сличить свои измерения между собой.
Т.е. пока сличают все изменится.
И еще то, что нарушается одновременность измерений. Опять-же сложно
договорится об одновременности измерений.
Вобщем как-то так.

Ну в этом вся и фишка. Наблюдатели из разных деревень, определяя центр Земли, могут вообще пренебречь временем. И пользоваться только тремя пространственными координатами. Наблюдатели в разных точках Вселенной, удаленных друг от друга, в принципе не могут пренебречь временем. И оно неизбежно начинает выступать как четвертая координата. Вселенная описывается моделью расширяющейся трехмерной замкнутой поверхности в четырехмерном пространстве. Если Вы легко смиряетесь с тем, что у замкнутой расширяющейся двумерной поверхности в трехмерном пространстве (поверхность надуваемого воздушного шарика) нет центра расширения (хотя наблюдателю в любой точке кажется, что центр расширения именно тут у него), то сделайте еще один шаг - и смиритесь легко с отсутствием центра расширения в нашей реальной Вселенной.

igor_da_bari
11.11.2009, 16:18
А вот про это я не подумал. Поразмыслю.


Поразмысли. Еще вот предыдущий мой пост посмотри. А я продолжу писать отчет. И выйду из форума. Иначе меня уволят нахрен :)

DenKur
11.11.2009, 16:47
Принципиально, что наблюдатели еще не родились друг для друга. :D . Хотя гдето уже проскакивало что скорость распространения информации выше скорости света.

igor_da_bari
11.11.2009, 16:52
Принципиально, что наблюдатели еще не родились друг для друга. :D

... или уже умерли :D

Влад
11.11.2009, 17:35
Мона я со своим глупым вопросом тут сбоку постою?
Почему КС списывают на Доплеровский эффект? Что такое ДЭ представление имею, благодаря ему зарабатываю на жизть.

igor_da_bari
11.11.2009, 17:52
Почему КС списывают на Доплеровский эффект?

Потому что:

1) До сих пор никаких других разумных механизмов, на которые можно было бы "списать КС" не предложено. Приверженцы эфирной теории говорят о "торможении фотонов в эфире", но не могут внятно объяснить, куда при этом девается энергия. И, тем более, не могут объяснить откуда берется "синее смещение", которое имеет место быть для некоторых боле-мене близких галактик - разгоняются что ли фотоны в этом их эфире?!

2) Раз галактики удаляются, то эффект Допплера, приводящий к КС должен иметь место. А факт разлетания галактик за 90 лет (прошедших с экспериментального обнаружения КС) получил и другие подтверждения, никак с КС не связанные. Например наличие реликтового излучения, плотность и температуру которого заранее и очень точно предсказал Гамов исходя из модели "Большого Взрыва" . Или наблюдаемая крупномасштабная структура Вселенной, которая при численном моделировании, включающем расширение, именно такой и должна получаться. Или хим. состав Вселенной - он тоже зависит от того, каким было начало и как все развивалось. Он тоже с расширением "дружит". Соотношение наблюдаемого количества барионов и лептонов. И т.д. и т .п. То есть связь КС с эффектом Допплера, то есть с расширением Вселенной естественна и позволяет непротиворечиво связать большое количество независмых наблюдаемых фактов.

Щаз на меня напрыгнет кто-нибудь из эфирщиков, но я сразу предупреждаю, что буду молчать, как рыба - ну их всех нафех...

Что такое ДЭ представление имею, благодаря ему зарабатываю на жизть.

А я вот представления не имею, что такое ДЭ, поскольку благодаря ему не зарабатываю на жизть.

Ar-Gen-Tum
11.11.2009, 18:25
...
Или наблюдаемая крупномасштабная структура Вселенной, которая при численном моделировании, включающем расширение, именно такой и должна получаться.
...

Вот угадайте, что-бы сказали, если-бы результаты моделирования не совпали с наблюдением?
Моделирование - эт фигня.
Да сейчас моделируют поведения некоторого ядерного(термоядерного) устройства.
Но, до этого было проведено немеряное количество тестов в 3-х средах.
И то читал, что суперкомпьютер считает модель 49 дней,
тогда как реальный процесс протекает за доли и единицы микросекунд.
Сколько должен длится расчет модели Вселенной, если самой Вселенной
не менее 10 млрд. лет??? :)

igor_da_bari
11.11.2009, 18:28
Вот угадайте, что-бы сказали, если-бы результаты моделирования не совпали с наблюдением?
Моделирование - эт фигня.
Да сейчас моделируют поведения некоторого ядерного(термоядерного) устройства.
Но, до этого было проведено немеряное количество тестов в 3-х средах.
И то читал, что суперкомпьютер считает модель 49 дней,
тогда как реальный процесс протекает за доли и единицы микросекунд.
Сколько должен длится расчет модели Вселенной, если самой Вселенной
не менее 10 млрд. лет??? :)

Вы когда-нибудь занимались численным моделированием? Если нет, давайте этот разговор оставим. Он совершенно бесполезен, как бесполезен разговор двух доярок об устройстве реактивного двигателя. Если да - на свой вопрос Вы сами сможете ответить.

megabobr
11.11.2009, 18:35
вот я думаю, если бы наши глаза воспринимали рентгеновские лучи, ночное небо показалось бы нам странным и незнакомым .ведь кванты рентгеновского излучения примерно в 1000 энергичнее квантов видимого света . Такие кванты возникают в астрофизической среде с высокой температурой и активно протекающими процессами .

igor_da_bari
11.11.2009, 18:39
Моделирование - эт фигня.


Вообще должен Вам сказать, что вот тут сказывается разница в философском подходе к жизни и научном.

Человек задал мне вопрос, я как мог на него ответил.

Откуда ни возьмись появились Вы, выхватили из моего ответа 1 (одну) фразу, слова в которой показались Вам знакомыми, и выдали свободную импровизацию на тему этой фразы без всяких выводов, но с налетом негатива. Причем импровизировали Вы на тему численого моделирования, о котором, судя по всему, знаете понаслышке из популярной прессы.

В результате дискуссия получила возможность уйти в любую сторону (что удобно Вам, потому что в любой стороне Вы найдете возможность найти знакомые Вам сочетания звуков и выдать очередную легковесную импровизацию с уходом в третью сторону), но никаких реальных дополнений к конкретному вопросу Влада не появилось. Это и есть то, что называется философией... :p

Влад
11.11.2009, 18:40
Щаз на меня напрыгнет кто-нибудь из эфирщиков,
Не-не-не, меня в их ряды не пиши, теорий строить не стану. Что такое Доплеровский Эффект представления не имеешь? Всё, я пошёл из темы. Я ни на кого не нападаю, я никого не защищаю, у меня нет никаких фактов. Я полностью согласен с тем, что КС помогает рассчитать расстояние до объектов. И чем оно больше, тем удалённей объект. Но не вяжется в моей разгильдяйской голове тот простой факт, что на фотоны не действует торможение. Ну не верю я в это. Не верю. Доказательств нет, фактов нет. Спорить не стану.

Ar-Gen-Tum
11.11.2009, 18:51
Вообще должен Вам сказать, что вот тут сказывается разница в философском подходе к жизни и научном.

Человек задал мне вопрос, я как мог на него ответил.

Откуда ни возьмись появились Вы, выхватили из моего ответа 1 (одну) фразу, слова в которой показались Вам знакомыми, и выдали свободную импровизацию на тему этой фразы без всяких выводов, но с налетом негатива. Причем импровизировали Вы на тему численого моделирования, о котором, судя по всему, знаете понаслышке из популярной прессы.
...

Вы совершенно точно угадали, именно "с налетом негатива". Хотя скорее с ухмылкой.
Верите в чудеса моделирования? На здоровье. Вреда от этого,
как Вы иногда говорите, никакого нет.
Я то-же доверяю моделированию. Когда модель достаточно простая и однозначная.
Типа транзисторного каскада с общим эмиттером. :)
Кстати, Вы знаете сколько параметров в продвинутой модели биполярного транзистора?
До ... пояса.
Вселенная конечно-же куда как проще. :)

DenKur
11.11.2009, 19:53
Вы совершенно точно угадали, именно "с налетом негатива". Хотя скорее с ухмылкой.
Верите в чудеса моделирования? На здоровье. Вреда от этого,
как Вы иногда говорите, никакого нет.
Я то-же доверяю моделированию. Когда модель достаточно простая и однозначная.
Типа транзисторного каскада с общим эмиттером. :)
Кстати, Вы знаете сколько параметров в продвинутой модели биполярного транзистора?
До ... пояса.
Вселенная конечно-же куда как проще. :)
ну тут вы передергиваете. Все дело в размере моделируемой системы и выбранном маштабе моделирования. Если спустится при моделировании БВ до уровня отдельных атомов (эл. частиц корректнее) то просче создать новый БВ.:D

megabobr
11.11.2009, 20:03
при построении космологических моделей обычно исходят из представлений о том , что Вселенная является однородной .Наблюдения же говорят о противооложном : вещество сконденсировано в звёздах, звёзды обединяются в большие комплексы - галактики и галактические скопления . Например в нашей галактике в диффузном состоянии находится всего около 3% вещества.
Каким же образом происходят фрагментация вещества в одной Вселенной ? Как образовались галактические скопления и сами галактики?
PlEASE !!!:D ;) :D

Ar-Gen-Tum
11.11.2009, 21:19
при построении космологических моделей обычно исходят из представлений о том , что Вселенная является однородной .Наблюдения же говорят о противооложном : вещество сконденсировано в звёздах, звёзды обединяются в большие комплексы - галактики и галактические скопления . Например в нашей галактике в диффузном состоянии находится всего около 3% вещества.
Каким же образом происходят фрагментация вещества в одной Вселенной ? Как образовались галактические скопления и сами галактики?
PlEASE !!!:D ;) :D
Галактик много. И они всякие разные. Но у большинства есть схожие черты -
это наличие хорошо заметных рукавов. И рукава расположены довольно симметрично
относительно центра.
И сдесь как-бы напрашивается само собой предположение, что галактики
образовались путем выброса вещества из центральной области.
Симметрия выбросов говорит о соблюдении закона сохранения импульса.
Если исходить из теории БВ, то получается что сначала вещество сжалось
в множество комочков, а потом эти комочки опять взорвались выбросами
и превратились в наблюдаемые галактики.
Сложноватый однако получается процесс.

DenKur
Чего там передергивать. Все кто так или иначе что-то моделировали
знают о свойствах этого процесса. И есно о слабых сторонах.
Конечно можно упростить систему и промоделировать. Иногда это
действительно бывает полезно. А иногда годится разве что в отчет.
Можно например смоделировать двигатель на фотонной тяге и обратится
за выделением средств на его постройку. Только фиг кто даст.
А для отчета и показа результата широкой общественности вполне сойдет. :)
...
Вот вспомнил интересный рассказ про моделирование.
Нужно было написать программку, моделирующую поведение силиконового шарика.
Компания собралась за компом и за пару часов написали прожку.
Но им показалось странным поведение шарика. И они начали копаться в алгоритме.
Все вроде было нормально. И тут один сбегал и купил такой шарик.
И компания дружно обрадовалась когда увидели, что реальный шарик
ведет себя практически также как и модель.
Это хороший пример в сторону моделирования.

КентаVR
11.11.2009, 21:37
при построении космологических моделей обычно исходят из представлений о том , что Вселенная является однородной .Наблюдения же говорят о противооложном : вещество сконденсировано в звёздах, звёзды обединяются в большие комплексы - галактики и галактические скопления .
Если вы будете строить зависимость объема пресловутого воздушного шарика (но теперь самого настоящего) от давления внутри него (или наоборот), вам необязательно будет учитывать, что газ тоже на самом деле не однороден, вещество там сконцентрировано в атомы/молекулы, но больше всего - пустоты :)

thirtyseconds
12.11.2009, 00:07
Здесь много всего интересного было высказано, я хотел бы пару моментов уточнить. В космологии, красное смещение на больших расстояниях не объясняется эффектом доплера (если бы это действительно было так, то стало бы необходимым признать что наиболее далекие галактики удаляются от нас со скоростью больше чем скорость света) а расширением самого пространства.
На эту тему очень много обсуждений на параллельном форуме.

Относительное того что Аргентум говорит :
"Но у большинства есть схожие черты - это наличие хорошо заметных рукавов. И рукава расположены довольно симметрично относительно центра.
И сдесь как-бы напрашивается само собой предположение, что галактики образовались путем выброса вещества из центральной области."

Вот лично мне это представляется крайне сомнительным. Во-первых по поводу "большинства". Например в нашей локальной системе галактик, спиральных подавляющее "меньшинство" ( http://seds.org/messier/more/local ), а большинство является эллиптическими или неправильными.
Второе - о "выбросах" вещества из центра... Предположение мне опять же крайне сомнительным, поскольку тут кто-то вектора искал, так вот нет известных векторов скоростей, которые бы свидетельствовали о каком то истечении вещества из галактического центра. За исключением случаев так называемых активных галактик, но там истечение из центра происходит от полюсов, в направлении перпендикулярном к диску галактики. Да и не материя там истекает, по моему.

Общепринятая на данный момент теория объясняющая существования спиральных рукавов говорит о том что видимые нами рукава, это временные структуры, некая область подсвеченная т.н. волнами плотности. В пользу этой теории говорит например тот факт, что именно на на внутренних (а не внешних) имеются пылевые структуры, то есть области с повышенной плотностью пыли и газа, что приводит к активному звездообразованию в этих областях и как следствие к тому что именно эти области мы видим в виде спиральных рукавов. Пара картинок на эту тему :
http://images.astronet.ru/pubd/2005/10/01/0001208280/ngc613_eso_c33.jpg

http://wsyachina.narod.ru/astronomy/galaxy_6/1.jpg

Это не главный аргумент, просто то что лично мне запомнилось. Ключевые слова для гуглевания "галактические волны плотности"

Ar-Gen-Tum
12.11.2009, 00:31
Здесь много всего интересного было высказано, я хотел бы пару моментов уточнить. В космологии, красное смещение на больших расстояниях не объясняется эффектом доплера (если бы это действительно было так, то стало бы необходимым признать что наиболее далекие галактики удаляются от нас со скоростью больше чем скорость света) а расширением самого пространства.
На эту тему очень много обсуждений на параллельном форуме.

Либо генерацией нового пространства в галактиках из вещества.
(Идея из трилогии С.Снегова. Хотя она возможно и имеет хождение в узких кругах.)


...
И сдесь как-бы напрашивается само собой предположение, что галактики образовались путем выброса вещества из центральной области."

Вот лично мне это представляется крайне сомнительным. Во-первых по поводу "большинства". Например в нашей локальной системе галактик, спиральных подавляющее "меньшинство" ( http://seds.org/messier/more/local ), а большинство является эллиптическими или неправильными.

Эллиптические - бывшие спиральные. Но старые и следы спиральной
структуры уже стерлись(размазались).
Вот неправильные это да, выбиваются.
Но к примеру Магеллановы облака возможно обрывки рукавов МП.

Второе - о "выбросах" вещества из центра... Предположение мне опять же крайне сомнительным, поскольку тут кто-то вектора искал, так вот нет известных векторов скоростей, которые бы свидетельствовали о каком то истечении вещества из галактического центра. За исключением случаев так называемых активных галактик, но там истечение из центра происходит от полюсов, в направлении перпендикулярном к диску галактики. Да и не материя там истекает, по моему.

Подразумевается выброс в относительно короткий срок. Как взрыв.
Хотя есть и предположения о достаточно долговременном истечении вещества.
Где-то читал.


Общепринятая на данный момент теория объясняющая существования спиральных рукавов говорит о том что видимые нами рукава, это временные структуры, некая область подсвеченная т.н. волнами плотности.
...

Есть галактики, у которых рукава как косы(типа как рисуют у смерти).
Т.е. перемычка и короткий, отстающий/опережающий рукав.
На волну не очень походит.

megabobr
12.11.2009, 08:43
Спиральная структура характерна для большинства галактик , разбросаных по Вселенной . Но как возникают спирали? И почему этот спиральный узор сохраняется в процессе вращения галактики?

megabobr
12.11.2009, 08:45
Спиральная структура характерна для большинства галактик , разбросаных по Вселенной . Но как возникают спирали? И почему этот спиральный узор сохраняется в процессе вращения галактики?:)

Weirdie
12.11.2009, 09:12
Эллиптические - бывшие спиральные. Но старые и следы спиральной
структуры уже стерлись(размазались).
Такие утверждения неплохо было бы чем-нибудь подверждать. :)
Логически рассуждая, если у спиральной галактики стерлась ее структура, то она станет линзообразной, а не спиральной. И кстати, что делать с карликовыми эллиптическими галактиками? Их довольно много, а вот карликовых спиральных не припомню ни одной.

Что касается рукавов, как последствий выброса вещества из ядра. Во-первых, мне лично совершенно не понятен механизм этого действа. Во-вторых, придется объяснить, почему эти выбросы происходят несколькоми порциями за раз (обычно двумя), а не одной или, скажем, сплошным кольцом по всей длине "экватора" ядра. В случае истечения вещества через полюса наличие именно двух порций вполне логично, а вот при выбросе в плоскости галактики как-то непонятно.

По поводу Магеллановых облаков, как обрывков рукавов Млечного Пути. Рукава, в общем-то находятся в одной плоскости. А Магеллановы облака находятся к ней под довольно значительным углом. Почему? Кроме того, обрывки вроде как должны удалятся, раз уж они оторвались, а с этими по-моему всё наоборот происходит.

thirtyseconds
12.11.2009, 09:57
Вообще было бы интересно все-таки выяснить эволюционный путь различных форм галактик. Но боюсь что срок прошедший для этого слишком мал. Видимое различие форм объясняется скорей всего разными условиями возникновения или более поздними событиями катастрофического характера, нежели постепенной эволюцией.

thirtyseconds
12.11.2009, 10:02
Спиральная структура характерна для большинства галактик , разбросаных по Вселенной . Но как возникают спирали? И почему этот спиральный узор сохраняется в процессе вращения галактики?

Мегабобр, вот если бы таки погуглили "галактические волны плотности" то стало бы понятно как возникают и почему "сохраняется". Теория может и не верна, сейчас трудно сказать, но на заданные вопросы она отвечает.
Вот, например, здесь - http://wsyachina.narod.ru/astronomy/galaxy_6.html

Вообще кстати вот еще широкоизвестный факт, про который забывают - о существовании какой-то скрытой массы сделали предположение исходя в том числе из того факта, что скорости вращения звезд вокруг центра галактики практически постоянны для всего диска, что непосредственно у центра, что на периферии. Соответственно, если предположить что именно звезды образуют спираль, то при равенстве скоростей любая спираль должна была бы целиком распасться буквально за пару оборотов, около 500 миллионов лет.

EDIT : Вернее, конечно звезды спираль и образуют, но вращение спирали это не вращение звезд.

igor_da_bari
12.11.2009, 13:00
Не-не-не, меня в их ряды не пиши, теорий строить не стану. Что такое Доплеровский Эффект представления не имеешь? Всё, я пошёл из темы. Я ни на кого не нападаю, я никого не защищаю, у меня нет никаких фактов. Я полностью согласен с тем, что КС помогает рассчитать расстояние до объектов. И чем оно больше, тем удалённей объект. Но не вяжется в моей разгильдяйской голове тот простой факт, что на фотоны не действует торможение. Ну не верю я в это. Не верю. Доказательств нет, фактов нет. Спорить не стану.

1) А-а-а-аааааа... ДЭ = Допплеровский эффект? Тогда имею представление, конечно...

2) Влад, сначала нужно определить смысл слов "на фотоны действует торможение", а потом уже разговаривать о том, справедливы эти слова или нет. Для меня это звучит примерно также, как "не верю я, что укус сыру вечером не помеха". То есть не то, что неверно, а просто как-то, извини, бессмысленно. Что такое торможение?! Как оно может действовать на что-то вообще и на фотоны в частности?! Какие можно найти факты и доказательства "за" или "против", если непонятно о чем вообще идет речь?!

igor_da_bari
12.11.2009, 13:54
.

igor_da_bari
12.11.2009, 13:55
Верите в чудеса моделирования? На здоровье. Вреда от этого,
как Вы иногда говорите, никакого нет.

Спасибо за великодушное разрешение верить в чудеса. Однако к сожалению мне трудно им воспользоваться. Верить или не верить во что бы то ни было мне сложно. Я предпочитаю знать или не знать.

А знаю я вот что. Никаких чудес моделирование не показыает. Как не показывает никаких чудес крестовая отвертка. Это просто инструменты, предназначенные для определенных работ. Не больше, но и не меньше. И отвертка, и моделирование широко используются каждое в своей области и без них там было бы очень сложно. И то, и другое, при неумелом обращении производит дрянь и чушь, при умелом - дает хорошие прочные результаты. В пределах области своего применения, разумеется.

А все остальное - словеса и философия, то есть наделение предметов и явлений свойствами, которые в реальности им не присущи... "Чудо, вера...". Это философу нужно чудо, диалектика и твердая вера, чтобы гвоздь забить. Нормальные люди просто берут молоток и забивают. И все.

Еще я знаю, что Вы зачем-то искусно увели разговор с КС на моделирование, которое Вы по каким-то своим причинам терпеть не можете. И я страшно удивляюсь, зачем Вы это сделали. Вероятно, эффект Допплера Вы не любите еще больше :)

megabobr
12.11.2009, 14:26
Вопрос о происхождении галактик и квазаров является одним из наиболее сложных в астрономии . Это и не удивительно . Ведь весь многообразный мир галактик был открыт буквально в наши дни . Можно сказать что изучение стуктуры галактик, процессов , происходящих в них , лишь начинается. Поэтому любая из предложеных ныне схем эволюции галактик может в конечном итоге оказатся слишком упрощенной, не отражающей всего комплекса явлений , разыгрывающихся а безбрежном космическом пространстве. Но такова логика развития науки.

VAnaS
15.11.2009, 00:42
Вопрос о происхождении галактик и квазаров является одним из наиболее сложных в астрономии .
Но этот вопрос должен решаться не изолированно, а как элемент (составная часть) решения общей проблемы генезиса и динамики развития общей структуры Вселенной.

Andre_aka_Lilit
15.11.2009, 02:44
Вселенная описывается моделью расширяющейся трехмерной замкнутой поверхности в четырехмерном пространстве. Если Вы легко смиряетесь с тем, что у замкнутой расширяющейся двумерной поверхности в трехмерном пространстве (поверхность надуваемого воздушного шарика) нет центра расширения (хотя наблюдателю в любой точке кажется, что центр расширения именно тут у него), то сделайте еще один шаг - и смиритесь легко с отсутствием центра расширения в нашей реальной Вселенной.

А можно пояснить, пожалуйста, что значит трехмерная поверхность? И где вообще у поверхности третье измерение?

Ar-Gen-Tum
15.11.2009, 03:27
А можно пояснить, пожалуйста, что значит трехмерная поверхность? И где вообще у поверхности третье измерение?
Сдесь igor_da_bari просто сделал упрощение(понижение размерностей),
что-бы сработала аналогия, которую мы в состоянии представить.

Влад
15.11.2009, 04:27
Влад, сначала нужно определить смысл слов "на фотоны действует торможение"
Примерно так - фотон несёт в себе энергию. Чем она больше, тем короче волна излучения. Один фотон принадлежит к ультрафиолетовой части светового диапазона, другой к красной. С течением времени оба постепенно теряют часть своей первоначальной энергии и тем самым они перемещаются влево по спектру.
Ведь структура фотона неизвестна. Или есть точное описание? Если существует, интересно узнать - каким образом она была определена? Или это всего лишь модель?

M.Sergey
15.11.2009, 12:55
Вселенная описывается моделью расширяющейся трехмерной замкнутой поверхности в четырехмерном пространстве. Если Вы легко смиряетесь с тем, что у замкнутой расширяющейся двумерной поверхности в трехмерном пространстве (поверхность надуваемого воздушного шарика) нет центра расширения (хотя наблюдателю в любой точке кажется, что центр расширения именно тут у него), то сделайте еще один шаг - и смиритесь легко с отсутствием центра расширения в нашей реальной Вселенной.
Выходит если лететь из одной точки вселенной прямо, не меняя направление, то неминуемо вернешься в исходную точку?

Nekkar
15.11.2009, 13:01
Выходит если лететь из одной точки вселенной прямо, не меняя направление, то неминуемо вернешься в исходную точку?

Наверное если лететь всегда прямо, то направление прямо сохранится вечно, т.к "шарик" все расширяется и расширяется и и нам это расширение не догнать и не перегнать.

M.Sergey
15.11.2009, 13:26
Наверное если лететь всегда прямо, то направление прямо сохранится вечно, т.к "шарик" все расширяется и расширяется и и нам это расширение не догнать и не перегнать.
А что вселенная расширяется быстрее скорости света?

Влад
15.11.2009, 13:58
Друзья, вы куда собрались? Тут до Марса проблема добраться.

igor_da_bari
16.11.2009, 12:36
А можно пояснить, пожалуйста, что значит трехмерная поверхность? И где вообще у поверхности третье измерение?

Можно. Для 4-мерного мира 3-х мерный (в котором на 1 измерение меньше) можно (условно, разумеется) считать поверхностью. В этом контексте термин "поверхность" используют довольно часто. Так же как в нашем 3-мерном мире мы называем поверхностью 2-мерные объекты.

А вообще все это игра слов, от которой мало что зависит. Хотите - поверхностью называйте, хотите - пространством. Хотите - даже точкой. Оно от этого ни на микрон не изменится и останется тем, что оно есть :)

igor_da_bari
16.11.2009, 14:01
Примерно так - фотон несёт в себе энергию. Чем она больше, тем короче волна излучения. Один фотон принадлежит к ультрафиолетовой части светового диапазона, другой к красной. С течением времени оба постепенно теряют часть своей первоначальной энергии и тем самым они перемещаются влево по спектру.
Ведь структура фотона неизвестна. Или есть точное описание? Если существует, интересно узнать - каким образом она была определена? Или это всего лишь модель?

Значицца так... :)

Во-первых слова "фотон несет в себе энергию" не совсем точны. Точнее было бы сказать "фотону можно приписать некоторую энергию". Дело в том, что энергия - понятие очень абстрактное. И смысл его не вполне понятен. И уж во всяком случае энергия - это не вещество, не явление и вообще не что-то материальное. Просто по каким-то причинам посчитанная определенным образом величина (которую назвали энергией) в замкнутой системе остается неизменной, что бы с системой ни происходило. А почему оно так и какое фундаментальное свойство вселенной стоит за этим фактом - тут нет ясности, хотя есть много споров и морд на эту тему поразбито предостаточно :) С фактически нулевым результатом.

Во-вторых - с какого, извини меня, перепугу, фотоны должны "с течением времени постепенно терять энергию"? Полная энергия всегда сохраняется (это собственно, единственное свойство энергии, ради которого само понятие "энергия" и было введено в рассмотрение - см. выше). Поэтому чтобы потерять энергию, фотону нужно с чем-то провзаимодействовать. С чем-то таким, энергия чего в результате взаимодействия увеличится ровно настолько, насколько уменьшится энергия фотона. Для более энергичных фотонов (рентген, гамма) это "что-то" может быть электронами - тогда получится Комптон эффект. В классической теории и в этом случае потеря энергии невозможна, но квантовая механика это дело позволяет. А для фотонов видимого света Комптон-эффект мог бы наблюдаться только в случае, если было бы что-то с де-Бройлевской длиной волны много больше, чем у электрона (то есть с много меньшей массой). Ничего такого мы (по крайней мере пока) не знаем. Далее - увеличение энергии этого "что-то" должно как-то проявляться. Вот видим мы туманность "Северная Америка". Потому что ее Денеб подсвечивает. И она переизлучает. Сбрасывает переданную ей энергию. И это "что-то", которое энергию у фотонов берет - тоже должно вести себя в том же духе. А мы этого не видим.

Если говорить об интерпретации КРАСНОГО СМЕЩЕНИЯ торможеием фотонов, то возникают и другие вопросы. Причем сразу много. Например - за каким хреном профили линий поглощения не уширяются? Если вдуматься, то в рамках "гипотезы торможения" это означает следующее - ВСЕ ФОТОНЫ ВСЕХ ДЛИН ВОЛН ТЕРЯЮТ СТРОГО ОДИНАКОВУЮ ЭНЕРГИЮ НА ПУТИ К ЗЕМЛЕ. И это количество потерянной энергии зависит строго от расстояния. И больше ни от чего. Не говоря уж о том, что Господу богу пришлось бы волчком вертеться, чтобы реализовать такое "что-то", с чем фотоны могли бы взаимодействовать столь изощренно (да и то еще неизвестно - вышло бы у него это или нет), так ведь это еще и следующий вопрос рождает - а почему фотоны от некоторых галактик демонстрируют не красное, а - наоборот - синее смещение? Причем опять же все сразу, хором. С какой радости наше гипотетическое "что-то" у фотонов от одних объектов энергию забирает (причем аккуратненько так), а фотонам от других объектов (даже в том же направлении лежащих) энергию, наоборот, отдает? И тоже с аптекарской точностью, а не статистическим образом, как это наблюдается во всех известных взаимодействиях.

В общем - ничего не мешает, разумеется, гипотезу этой самой "нечты" развивать и пропагандировать. А так же верить в нее или не верить. Но я бы на месте ее адептов для начала сделал бы ее НЕПРОТИВОРЕЧИВОЙ. То есть отвечающей на вопросы, подобные тем, что перечислены выше. А потом сделал бы на основе этой гипотезы численное предсказание. И экспериментально это предсказание подтвердил бы.

А пока этого не произошло, меня вполне устраивает интерпретация КС с помощью эффекта Допплера. Поскольку она непротиворечиво объясняет всю совокупность фактов, связанных с КС. И поскольку этот эффект - не гипотетическое "нечто", а может быть воспроизведен в лаборатории. С численно предсказуемым результатом.

И в-третьих: структура фотона и в самом деле неизвестна. Скорее всего потому, что ее у него просто нет. Вообще фотон - не исключение. Не выявлено экспериментально до сих пор никаких структур ни у одного лептона. Скорее всего опять же потому, что нету у них никакой структуры. Да и с чего бы ей обязательно быть? Из чего следует обязательное наличие структуры у всего сущего? В любом случае отсутствие представлений о структуре фотона, электрона, мюона, тау-лептона, всех тре нейтрин не мешает знать их свойства достаточно хорошо, чтобы расчитывать количественные характеристики процессов, в которых они участвуют (в частности, потери энергии) и получать согласие таких расчетов с экспериментом.

igor_da_bari
16.11.2009, 14:04
Выходит если лететь из одной точки вселенной прямо, не меняя направление, то неминуемо вернешься в исходную точку?

А Вы попробуйте :)

Влад
16.11.2009, 14:28
с какого, извини меня, перепугу, фотоны должны "с течением времени постепенно терять энергию"?
На мой взгляд, фотоны на своём пути сталкиваются с ещё более мелкими частицами. Настолько мелкими, что те неспособны хоть как-то повлиять на вектор их скорости. Но эти частицы их чуть притормаживают. Настолько "чуть", что изменение становится заметным лишь на больших расстояниях.
а почему фотоны от некоторых галактик демонстрируют не красное, а - наоборот - синее смещение? Причем опять же все сразу, хором.
Ну зачем было сразу выкладывать такого козыря?! Бита моя карта, признаю, нечем крыть. Проигрывать умею.

igor_da_bari
16.11.2009, 14:34
На мой взгляд, фотоны на своём пути сталкиваются с ещё более мелкими частицами. Настолько мелкими, что те неспособны хоть как-то повлиять на вектор их скорости. Но эти частицы их чуть притормаживают. Настолько "чуть", что изменение становится заметным лишь на больших расстояниях.


Влад, в квантовой механики (на которую пока даже эфиристы не замахивались) невозможно сколь угодно малые передачи энергии и импульса. С классической точки зрения фотон вообще не может менять длину волны. С квантово-механической - может, НО (про это НО я уже писал в предыдущем посте).

thirtyseconds
16.11.2009, 14:49
Здесь вот опять же, позволю себе такую наглость, уточнить принципиальный для понимания, с моей точки зрения, момент. Упомянутый Игорем эффект доплера в данном случае объясняется не удалением самого объекта, а расширением пространства между нами и этим объектом.

Почему так ? Ну хотя бы потому что если бы мы наблюдали действительно скорости самих объектов, то в этом случае нам нужно было бы признать что мы находимся в центре вселенной. Так как в какую сторону не погляди мы видим убегающие от нас галактики.

Влад
16.11.2009, 15:01
Э...э...э, я это, я не начинаю строить теорию. Но интересно. Взять один объект, например автомобиль, в 2 момента времени. В первом случае он стоит, во втором движется с какой-то скоростью. Имеем вроде бы один и тот же автомобиль. Но во втором случае он обладает энергией. Чем отличается структура или химический состав? В какой части авто содержится энергия? В передней и тянет его вперёд? Или в задней и толкает?
Это вопросы в никуда, просто мысли.

КентаVR
16.11.2009, 15:06
В какой части авто содержится энергия?
Энергия, которая толкает его вперед, содержится в бензобаке :)

Упомянутый Игорем эффект доплера в данном случае объясняется не удалением самого объекта, а расширением пространства между нами и этим объектом.
А когда машина от нас и к нам едет и "звучит" по-разному - это тоже от расширения пространства?

M.Sergey
16.11.2009, 15:07
Вселенная описывается моделью расширяющейся трехмерной замкнутой поверхности в четырехмерном пространстве.
Если принять за основу многомерность вселенной, то зачем тогда придумывать всякие «темные материи», «темные энергии»? Логично предположить, что существует материя и энергия четвертого измерения, которого мы не можем увидеть, но можем зафиксировать его воздействие.

thirtyseconds
16.11.2009, 15:09
А когда машина от нас и к нам едет и "звучит" по-разному - это тоже от расширения пространства?

Нет, конечно.

У вас какие то возражения по сути ? :)

M.Sergey
16.11.2009, 15:11
Энергия, которая толкает его вперед, содержится в бензобаке :)
В бензобаке содержится бензин, а энергия выделяется при сжигании этого бензина.

thirtyseconds
16.11.2009, 15:16
Если принять за основу многомерность вселенной, то зачем тогда придумывать всякие «темные материи», «темные энергии»?

Темная материя это попытка объяснить расхождение между видимым движением некоторых объектов и наблюдаемой массы, которой не достаточно что бы эти самые объекты двигались так как они двигаются - звезды вокруг центра галактик, скопления и великий аттрактор и т.д.

igor_da_bari
16.11.2009, 15:17
Если принять за основу многомерность вселенной, то зачем тогда придумывать всякие «темные материи», «темные энергии»? Логично предположить, что существует материя и энергия четвертого измерения, которого мы не можем увидеть, но можем зафиксировать его воздействие.

Четвертое измерение - это просто время.

Влад
16.11.2009, 15:20
Энергия, которая толкает его вперед, содержится в бензобаке
Несогласен. Разогнать авто на строго горизонтальном участке и дороги, выключить двигатель и перейти на нейтральную передачу. Как в этом случае? В чём та сила инерции? :)

КентаVR
16.11.2009, 15:23
Нет, конечно.

У вас какие то возражения по сути ? :)

Нет, возражений теперь не имею, наверное, я не вполне точно понял вашу мысль.

thirtyseconds
16.11.2009, 15:26
Нет, возражений теперь не имею, наверное, я не вполне точно понял вашу мысль.

Я вот по этому и посчитал что нужно уточнить. Вот еще можно вики процитировать :

"Сущность явления

Часто космологическое красное смещение связывают с эффектом Доплера. Однако, на самом деле, эффект Доплера не имеет никакого отношения к космологическому красному смещению, которое в действительности связано с расширением пространства согласно ОТО. В наблюдаемое красное смещение от галактик вносит вклад как космологическое красное смещение из-за расширения пространства Вселенной, так и красное или фиолетовое смещения эффекта Доплера вследствие собственного движения галактик. При этом на больших расстояниях вклад космологического красного смещения становится преобладающим[1].

Образование космологического красного смещения можно представить так: рассмотрим свет — электромагнитную волну, идущую от далёкой галактики. В то время как свет летит через космос, пространство расширяется. Вместе с ним расширяется и волновой пакет. Соответственно, изменяется и длина волны. Если за время полёта света пространство расширилось в два раза, то и длина волны и волновой пакет увеличивается в два раза."

M.Sergey
16.11.2009, 15:27
Четвертое измерение - это просто время.
А если все-таки есть материальное четвертое измерение, не доступное для наблюдений? Так же как не доступно третье, для гипотетического наблюдателя живущего в двухмерном пространстве.

КентаVR
16.11.2009, 15:34
Несогласен. Разогнать авто на строго горизонтальном участке и дороги, выключить двигатель и перейти на нейтральную передачу. Как в этом случае? В чём та сила инерции? :)
А чего тут непонятного? Т.н. "химическая" энергия, носителем которой являются молекулы горючего (которые, кстати, находятся в бензобаке) превращается в "тепловую", "тепловая" - в "механическую", механическая по трансмиссии передается на колеса, машина едет. Если отключить двигатель, она будет по инерции катиться дальше, растрачивая накопленную кинетическую энергию на сопротивление трению, воздуху и т.д.

Можно, конечно, и без бензина заставлять машины двигаться, если, как говорят на армянском радио, пускать их с горы Арарат.

igor_da_bari
16.11.2009, 15:48
Здесь вот опять же, позволю себе такую наглость, уточнить принципиальный для понимания, с моей точки зрения, момент. Упомянутый Игорем эффект доплера в данном случае объясняется не удалением самого объекта, а расширением пространства между нами и этим объектом.


Таки нет :)

Drago
16.11.2009, 15:51
Нету центра, он везде :)


эта всё глупости. центр - там где я.
"где я-там успех!" (ц) Покровский.
и вообще, вы все на самом деле несуществуете, так как сия вселенная есть плод моего воображения...
мощно это я задвинул, да?
вроде, поговаривают, что такую точку зрения логикой опровергнуть нельзя..?

igor_da_bari
16.11.2009, 15:53
А если все-таки есть материальное четвертое измерение, не доступное для наблюдений? Так же как не доступно третье, для гипотетического наблюдателя живущего в двухмерном пространстве.

Сергей... Ну - если есть, то есть. Как можно ответить на вопрос "Что, если есть что-то, что в принципе ненаблюдаемо никаким каком?". Я лично думаю, что ненаблюдаемость ЧЕГО-ТО в принципе эквивалентно отсутствию этого ЧЕГО-ТО.

Вообще-то было показано (не помню, кем и когда), что только во Вселенной с 3 пространственными измерениями возможно существование стабильной материи.

thirtyseconds
16.11.2009, 15:55
Игорь, а подробней ?

Понятно, что эффект Доплера является одним из компонентов космологического смещения, но на больших расстояниях им можно пренебречь.

Кроме этого увеличение длины волны конечно похоже на эффект Доплера и периодически когда говорят о космологическом смещении так и говорят - эффект Доплера. Но важно то что природа увеличения длины волны у фотонов приходящих к нам с больших расстояний не связана со скоростью самого объекта.

M.Sergey
16.11.2009, 16:03
Как можно ответить на вопрос "Что, если есть что-то, что в принципе ненаблюдаемо никаким каком?". Я лично думаю, что ненаблюдаемость ЧЕГО-ТО в принципе эквивалентно отсутствию этого ЧЕГО-ТО.
А как тогда быть с «темной материей», которую невозможно увидеть? Есть она или нет?

Andre_aka_Lilit
16.11.2009, 16:07
В бензобаке содержится бензин, а энергия выделяется при сжигании этого бензина.

Тогда как, энергия не существует сама по себе, а привязана исключительно к процессу ее получения? :)

ZamaZzZka
16.11.2009, 16:26
А как тогда быть с «темной материей», которую невозможно увидеть? Есть она или нет?
Ну мы же наблюдаем ее косвенно, в отличие от приведенного вами примера :)

Ar-Gen-Tum
16.11.2009, 16:29
Не может фотон менять длину волны.
Ибо физически нет у него этой характеристики.
Если в металлическом прутке начать двигать заряд туда-сюда,
то от прутка в пространстве будет распространятся эл.-магн. волна.
Эта волна, обладает свойством убывать обратно-пропорционально
с квадратом расстояния от источника(когда источник близок к точечному).
Свет также обладает этим свойством, но по другой причине.
Представьте пулемет, который стреляет очередью, при этом вращаясь.
Плотность пуль убывает обратно-пропорционально квадрату расстояния от пулемета, но убойная сила сохраняется на всем протяжении полета,
если пренебречь торможением.
Так и фотон больше похож на пулю, но в чем-то пожож и на волну.
Кстати пуля также обладает волновыми свойствами - т.н. волна Де-Бройля.

M.Sergey
16.11.2009, 16:38
Ну мы же наблюдаем ее косвенно, в отличие от приведенного вами примера :)
Да ни кто, ни чего не наблюдает! Этот термин ввели, чтобы объяснить то, что пока не поддается объяснению.

thirtyseconds
16.11.2009, 16:52
Аргентум, так он ее и не меняет. Я наверно не точно опять высказался.

Тут что бы не запутаться я еще раз. Есть эффект Доплера с которым каждый знаком на бытовом уровне, когда например звук объекта, который имеет какую то скорость относительно нас, изменяет частоту.

Но то смещение линий в красную сторону, которое наблюдается в спектрах наиболее удаленных галактик не связано со скоростью самого объекта относительно нас (хотя конечно эффект доплера может вносить свой вклад)

Насколько я понимаю такая точка зрения является общепринятой. Для меня главный аргумент в ее пользу, опять же повторю, это что если бы галактики действительно имели бы те скорости, которые мы наблюдаем, то мы оказались бы в центре вселенной. То есть оказалось бы что все объекты во вселенной удаляются от нас.

Игорь высказался коротко, вроде бы против такой точки зрения, может быть я действительно что-то важное упускаю или что-то не точно изложил...

igor_da_bari
16.11.2009, 16:53
А как тогда быть с «темной материей», которую невозможно увидеть? Есть она или нет?

"Ненаблюдаемо" и "нельзя увидеть" - это разные вещи, согласитесь. "Ненаблюдаемо" означает полное отсутствие какого-либо взаимодействия с наблюдаемым миром. "Нельзя увидеть" означает отсутствие достаточной интенсивности излучения в видимом диапазоне - и все. Электрический ток тоже ведь увидеть нельзя. Так же как магнитное поле, нейтрон, протон, пи-мезон. Тем не менее по их взаимодействию с чем-то наблюдаемым можно надежно определить, что все это существует.

Что касается "темной материи" в первоначальном смысле - она как раз и возникла из наблюдаемости. Стало ясно, что есть чего-то много, что притягивает. Из наблюдаемых фактов стало ясно, а не из игры ума. А больше никак не проявляется. Глазом - да, не видно. Ну и что?

igor_da_bari
16.11.2009, 16:58
Понятно, что эффект Доплера является одним из компонентов космологического смещения, но на больших расстояниях им можно пренебречь.

Кроме этого увеличение длины волны конечно похоже на эффект Доплера и периодически когда говорят о космологическом смещении так и говорят - эффект Доплера. Но важно то что природа увеличения длины волны у фотонов приходящих к нам с больших расстояний не связана со скоростью самого объекта.

Может, я чего упустил, но еще 4 года назад, когда я в последний раз был физиком, увеличение длины волны у фотонов, приходящих к нам с космологических расстояний, таки было связано со скоростью объекта, который эти фотоны излучил.

igor_da_bari
16.11.2009, 17:06
Не может фотон менять длину волны.


Эффект Комптона как тогда объясните?

Ar-Gen-Tum
16.11.2009, 17:49
Эффект Комптона как тогда объясните?
На сколько я понял этот эффект обнаружен не для всех энергий излучения.
А как-бы только для рентгеновского спектра.
Вероятно, при взаимодействии фотона с электроном происходит перераспределение
энергии между фотоном и электроном. Но первичный фотон уничтожается,
а потом уже излучается другой фотон.
Кстати вычитал, что это справедливо для свободных электронов.
Кстати вот тоже интересно:
Электроны в ЭЛТ(CRT) движутся в узком пучке. И при этом особо
не стремятся разлететься веером(конусом) из-за немалых сил
электростатического отталкивания.
...
thirtyseconds
(Кстати, лучше-б Вы при обращении ник правильно писали, дабы было понятно
к кому направлено обращение.)
Если пространство может расширятся, то вероятно может и сжиматся.
А раз может расширятся и сжиматся, то может и колебатся.
Т.е. возможны волны пространства. С весьма значительными периодами.
И возможно мы живем в такую эпоху, что проваливаемся в пространстве
отчего кажется, что остальное пространство расширяется.
Это я к чему собсно. Если некоторый объект излучил фотон определенной энергии,
то эта энергия не может куда-то исчезнуть.
А если фотон покраснел, то часть энергии куда-то делась.
(Если мимо проносится гудящая электричка, то звук сначала высокий,
а потом меняется на низкий. И в среднем остается постоянным.)
Так вот если наша часть мира проваливается во впадину волны,
то часть энергии фотона как раз и запасается в этом проваливании.
А потом пойдем на гребень и энергия будет отдаваться.
А в сумме останется постоянной.
Сорри, несколько сумбурно получилось, но вероятно смысл понятен.

M.Sergey
16.11.2009, 19:27
"Ненаблюдаемо" и "нельзя увидеть" - это разные вещи, согласитесь. "Ненаблюдаемо" означает полное отсутствие какого-либо взаимодействия с наблюдаемым миром.
Конечно, под словом «увидеть» подразумевал более широкий смысл; не только визуальные наблюдения, но и с помощью различных приборов.
Но давайте снова представим наблюдателя в двухмерном пространстве. Находясь на поверхности сферы, он может наблюдать, как расширяется его вселенная, гравитационное воздействие некой огромной массы, но установить, что в этот момент происходит закачка газа в сферу, он не сможет. Разве это не похоже на то, что мы наблюдаем в своей вселенной?

thirtyseconds
16.11.2009, 19:53
Может, я чего упустил, но еще 4 года назад, когда я в последний раз был физиком, увеличение длины волны у фотонов, приходящих к нам с космологических расстояний, таки было связано со скоростью объекта, который эти фотоны излучил.

Игорь, вы же понимаете, я не свою теорию альтернативную излагаю... А ориентируюсь на прочитанное. И поскольку вопрос на мой взгляд важный, хотел бы уточнить. Как вот тогда можно понимать следующее :

http://astronet.ru/db/msg/1162269
"Космологическое красное смещение связано с удаление от нас наблюдаемых объектов из-за расширения Вселенной. У наиболее далеких галактик и квазаров наблюдается К.С. http://images.astronet.ru/pubd/2000/12/19/0001162269/tex/formula7.gif. При этом в видимой области спектра наблюдаются линии, далекого ультрафиолетового диапазона." (автор заметки Сурдин)

Или вот из учебника по астрономии Кононовича (2009 г.)

"Космологическое красное смещение заключается в том, что длины волн излучения далеких галактик меняются со временем пропорционально изменению масштабного фактора ... "

Или вот http://cosmo.irk.ru/part2-6.html :

"Длина волны света, путешествующего во внегалактическом пространстве, также подчиняется соотношению (2.6) (см. рис.2.6.1). Поэтому если испущенная галактикой в момент времени t длина волны есть λ, то длина волны принятого нами света окажется равной λ'=λ/a(t). Подставив это значение в формулу (2.1), получаем:
http://cosmo.irk.ru/frml/2_7.gif (2.7)

Обратим внимание: при выводе этой формулы мы нигде не используя понятие скорости удаления галактики-источника, оперируя только понятием расширения пространства. Можно показать, что в состав красного смещения, определяемого по формуле (2.7), входит и эффект Доплера (причем с учетом релятивистских поправок), и замедление времени в гравитационном поле, предсказываемое общей теорией относительности Эйнштейна."


Ошибиться в понимании последнего абзаца и приведенной формулы сложно. Красное смещение удаленных объектов определяется, в целом, т.н. масштабным фактором вселенной.

thirtyseconds
16.11.2009, 19:57
Это я к чему собсно. Если некоторый объект излучил фотон определенной энергии, то эта энергия не может куда-то исчезнуть.
А если фотон покраснел, то часть энергии куда-то делась.

Нет, Ar-Gen-Tum. Вот, сверху я несколько цитат привел. Из них должно быть очевидно, что сам фотон энергии не теряет.

Ar-Gen-Tum
16.11.2009, 21:16
Нет, Ar-Gen-Tum. Вот, сверху я несколько цитат привел. Из них должно быть очевидно, что сам фотон энергии не теряет.
Ну какже очевидно-то.
Вот посудите - здесь на Земле мы видим, что спектр Солнца такой-то,
а наблюдатель в удаленной галактике видит, что спектр Солнца покраснел.
Но фотоны-то одинаковые. Только одни попадают на наши приборы,
а другие проходят мимо и попадают на приборы далекого исследователя.
Но раз они покраснели, то значит часть энергии потеряли. Это-то как раз и очевидно.
Вот куда они потеряли часть энергии? Вы-же и сами говорите, что они энергию никуда не теряли.
Вот электричка несется и гудит. Для наблюдателя сзади звук ниже,
для наблюдателя на электричке такой как есть, для наблюдателя впереди выше.
Если вычислить среднее, то оно будет такое как есть.
А Вселенная расширяется для всех наблюдателей и фотоны конкретно теряют энергию.
(Давайте скажите, что энергия и тратится на расширение. И чем быстрее
расширение, тем больше ее и тратится. Такая вот вселенская ПОС. :) )

thirtyseconds
16.11.2009, 22:27
Тут вполне возможно, что я ошибаюсь или просто что-то недопонимаю. Теряют энергию фотоны, не теряют, в состоянии они ее терять или нет и при каких условиях :)

Главное, что я пытаюсь сказать, основной пункт по которому я тут вообще выступаю, так это то что согласно общепринятой (на данный момент) точки зрения космологическое красное смещение объясняется не перемещением объектов относительно нас, а расширением самого пространства. Или, иначе говоря, как было процитировано выше, масштабным фактором вселенной.

С моей точки зрения это очень важно, поскольку если заменить такое объяснение на эффект доплера, то получается совершенно иная картина вселенной, отличная от той, что обсуждается и фигурирует в современных материалах.

КентаVR
16.11.2009, 23:00
Так а расширение пространства разве не приводит к увеличению расстояния от нас до галактик?

Andre_aka_Lilit
17.11.2009, 00:12
Господа, может я чушь гороДю, но с точки здравого смысла, если ежесекундно расстояние между галактиками увеличивается вдвое, то фотон ежесекундно должен вдвое терять энергию (или его длина волны должна увеличиваться вдвое). Правильно? Но тогда чушь получается :( мы ближайшую галактику вообще видеть не должны??? А в случае с туманностью Андромеды, так фотоны должны превысить вдвое скорость света в первую же секунду!!! Ну судя по расстояниям :(:(

КентаVR
17.11.2009, 00:19
Господа, может я чушь гороДю, но с точки здравого смысла, если ежесекундно расстояние между галактиками увеличивается вдвое, то фотон ежесекундно должен вдвое терять энергию (или его длина волны должна увеличиваться вдвое). Правильно? Но тогда чушь получается :( мы ближайшую галактику вообще видеть не должны??? А в случае с туманностью Андромеды, так фотоны должны превысить вдвое скорость света в первую же секунду!!! Ну судя по расстояниям :(:(
А вы школу закончили?

Nekkar
17.11.2009, 00:33
"Ненаблюдаемо" и "нельзя увидеть" - это разные вещи, согласитесь. "Ненаблюдаемо" означает полное отсутствие какого-либо взаимодействия с наблюдаемым миром. "Нельзя увидеть" означает отсутствие достаточной интенсивности излучения в видимом диапазоне - и все. Электрический ток тоже ведь увидеть нельзя. Так же как магнитное поле, нейтрон, протон, пи-мезон. Тем не менее по их взаимодействию с чем-то наблюдаемым можно надежно определить, что все это существует.


Кстати забыл спросить, вы Кесаря все же с рогаткой и гвоздиками поймали? Что-то вспомнился он именно на этом посте, странно что его тут еще нет.

Ar-Gen-Tum
17.11.2009, 00:36
Поразмышляв за рюмочкой чая вот подумал -
а собсно почему-бы Земле не находится пусть не в самом центре видимой Вселенной,
но хотя-бы не далеко от центра?
Какому закону это могло-бы противоречить?
А если это не противоречит физ. законам, то зачем упираться
и пытаться доказывать, что Земля находится не в центре, а в самой ... ? :)
Ведь наблюдения показывают объекты со всех сторон.
:)

Nekkar
17.11.2009, 00:42
Чтобы говорить о центре, нужно сначала найти край. А его пока не найдено. Граница ВИДИМОЙ части вселенной еще не факт что является границей вселенной вообще. Да и не факт что эта граница существует в природе. Даже скорее наоборот, хотя кто ее знает

Юрий
17.11.2009, 00:48
Чтобы говорить о центре, нужно сначала найти край. А его пока не найдено. Граница ВИДИМОЙ части вселенной еще не факт что является границей вселенной вообще. Да и не факт что эта граница существует в природе. Даже скорее наоборот, хотя кто ее знает
Мдаа... А ведь так и умрем с такими вот вопросами. Обидно :(.

Помню дело было когда я учился в третьем классе, к нам на урок пришли несколько учеников из одиннадцатого класса, причем не просто какие то ученики, а особенные ботаны, отобранные завучем (короче говоря, для нас третиклассников, они были просто ходячей энциклопедией :)).
Учитель нам и говорит: "Ребята, у нас сегодня такая вот замута в школе, вот эти старшеклассники ходят по классам и отвечают на любые ваши вопросы. Спрашивайте". Я обрадовался :p (наивный был), думаю, что сейчас узнаю ответ на самый интересующий меня вопрос: "Где кончается Вселенная!". Подошла моя очередь задать вопрос, я спросил, а в ответ скромная задумчивая пауза, пожимание плечами и... ничего больше :(. Двадцать лет прошло, но ничего не изменилось, все те же вопросы, все те же рассуждения.

Andre_aka_Lilit
17.11.2009, 00:49
А вы школу закончили?

Не заканчивал я школу ;)O:) Извините, но так уж случилось :( Я честно больше не буду.:D

Просто в параллельной теме было:
.........................
Очень грубо - если до объекта А у нас одна условная единица расстояния, а до объекта Б 10 и если за единицу времени пространство расширяется в два раза, то через одну единицу времени до объекта А будет 2 ед, а до объекта Б уже 20. То есть для нас скорость А = 1, а скорость Б в 10 раз больше (то что мы и наблюдаем - чем дальше объект, тем больше скорость убегания, Закон Хаббла). И если скорость света в такой системе скажем 3 ед раст. за единицу времени то от объекта А мы получим фотон, а от объекта Б никогда. И чем дольше будет идти к нам фотон тем дальше он будет находиться от нас. Через две единицы времени расстояние будет 17 далее 31, 59 и т.д.
...............

Тем более что я извинился и предупредил про чушь. :) И еще тем более, что я гуманитарий :D Но действительно хочется понять такие глобальные вещи.;)O:)

Viacheslav
17.11.2009, 00:53
А кто сказал,что скорость расширения одинакова во всём пространстве? У неё только на границе верхнее ограничение с . А в центре - ноль :D

КентаVR
17.11.2009, 01:01
Ого, сам управдом снизошел до обсуждения пространных вопросов.
Andre_aka_Lilit, а по какой дисциплине гуманитарий? Просто касательно предыдущих вопросов - "если бы у бабушки были определенные половые признаки..." и так далее.

Andre_aka_Lilit
17.11.2009, 01:04
Ого, сам управдом снизошел до обсуждения пространных вопросов.
Andre_aka_Lilit, а по какой дисциплине гуманитарий? Просто касательно предыдущих вопросов - "если бы у бабушки были определенные половые признаки..." и так далее.

Лоеры мы....:D Харьковская юрка имени очень мудрого Ярослава:D:D
Кстати, про бабушку не писал, вроде, тем более про признаки.......:D

Viacheslav
17.11.2009, 01:09
Дык Управдом вроде не бабушка :D. Вот и периодически захаживает...золочёное брюхо поглаживает :D

КентаVR
17.11.2009, 01:11
Andre_aka_Lilit, Про бабушку высказался один весьма популярный в России гражданин, а данное сравнение было применено мной к фразе "если ежесекундно расстояние между галактиками увеличивается вдвое"

Andre_aka_Lilit
17.11.2009, 01:17
Andre_aka_Lilit, Про бабушку высказался один весьма популярный в России гражданин, а данное сравнение было применено мной к фразе "если ежесекундно расстояние между галактиками увеличивается вдвое"

Это понятно, но все равно хотелось бы понять "В чем сила, брат?":D

thirtyseconds
17.11.2009, 01:26
Поразмышляв за рюмочкой чая вот подумал -
а собсно почему-бы Земле не находится пусть не в самом центре видимой Вселенной,
но хотя-бы не далеко от центра?
Какому закону это могло-бы противоречить?


Ar-Gen-Tum, насколько я понимаю, появление выделенной системы отсчета приводит к нарушению принципа относительности... Хотя таковой системой в любом случае уже является реликтовый фон.

Да и потом это массу вопросов ставит, да и вообще с теорией большого взрыва и с реликтовым излучением можно завершать (потому что, как мне это представляется должны наблюдаться какие-то отличия в температуре фона, по отношению к "центру" вселенной, чего не наблюдается). Ну и плюс чисто философские неувязки... С какой стати наш Млечный путь, который не обладает никакими другими известными уникальными характеристиками, должен находиться в центре вселенной ?


Двадцать лет прошло, но ничего не изменилось, все те же вопросы, все те же рассуждения.


Юрий, изменилось очень многое. Космология именно в эти годы переживает свои золотые годы, которые начались с установления факта что вселенная расширяется с ускорением (это всего 11 лет назад произошло). А то что мы по прежнему эти вопросы задаем, свидетельствует лишь о степени интереса к этой науке. Я ссылки на учебники уже раза три приводил. Уверен, что никто даже не заглянул (да и я сам читал, только когда хотел уточнить что-то, что бы не промахнуться в запале). А это ведь науч-поп. Облегченное изложение для начинающих гуманитариев.

thirtyseconds
17.11.2009, 01:31
Но действительно хочется понять такие глобальные вещи.;)O:)

Цифры были взяты абсолютно от балды, что бы облегчить пример.

Andre_aka_Lilit
17.11.2009, 01:35
Я ссылки на учебники уже раза три приводил. Уверен, что никто даже не заглянул (да и я сам читал, только когда хотел уточнить что-то, что бы не промахнуться в запале). А это ведь науч-поп. Облегченное изложение для начинающих гуманитариев.

Цифры были взяты абсолютно от балды, что бы облегчить пример.


:oops::oops::oops::oops::oops:

Ar-Gen-Tum
17.11.2009, 02:05
Ar-Gen-Tum, насколько я понимаю, появление выделенной системы отсчета приводит к нарушению принципа относительности... Хотя таковой системой в любом случае уже является реликтовый фон.
...

Реликтовый фон?
Действительно интересно. Ведь если со всех сторон точно измерить
его спектр, то можно вычислить абсолютный вектор скорости Земли.
Рели́ктовое излуче́ние (или космическое микроволновое фоновое излучение от англ. cosmic microwave background radiation)[1] — космическое электромагнитное излучение с высокой степенью изотропности
Т. е. если Земля никуда не движется относительно реликтового излучения,
то значит она находится в центре.
А если таки движется, то должен обнаружится эффект Доплера-Физо
по отношению к реликтовому излучению.
:)

thirtyseconds
17.11.2009, 03:02
Уже измерили, таки движется :) - http://www.astronet.ru/db/msg/1170612/node61.html

"Еще в 1969 г. была измерена дипольная анизотропия реликтового фона, связанная с движением Земли относительно изотропного реликтового излучения. Измеренное значение дипольной анизотропии http://images.astronet.ru/pubd/2002/05/14/0001176797/img1658.gif мК соответствует (по формуле эффекта Допплера, http://images.astronet.ru/pubd/2002/05/14/0001176797/img1659.gif) скорости движения барицентра Солнечной системы со скоростью http://images.astronet.ru/pubd/2002/05/14/0001176797/img1660.gif км/с. Учитывая движение Солнечной системы относительно центра Галактики, определена скорость движения местной группы галактик относительно реликтового фона http://images.astronet.ru/pubd/2002/05/14/0001176797/img1661.gif км/с. После 30 лет исследований анизотропии реликтового фона в эксепериментах "Реликт" (Россия) и COBE (США) измерены амплитуды флюктуаций температуры реликтового излучения в угловых масштабах несколько 10 градусов на уровне 30 мкК ( http://images.astronet.ru/pubd/2002/05/14/0001176797/img1662.gif)."

igor_da_bari
17.11.2009, 13:05
Кстати забыл спросить, вы Кесаря все же с рогаткой и гвоздиками поймали? Что-то вспомнился он именно на этом посте, странно что его тут еще нет.

Кесаря Борис транклюкировал нахрен. O:)

igor_da_bari
17.11.2009, 13:45
На сколько я понял этот эффект обнаружен не для всех энергий излучения.
А как-бы только для рентгеновского спектра.

Геннадий, не уводите разговор в сторону. Вы высказали утверждение, что у фотона не может быть длины волны, поскольку он не волна. А стало быть, менять он ее не может. Я Вам возразил, а Вы мне отвечаете про энергию что-то.

Ну, и в самом деле, Комптон эффект, меняющий длину волны фотона, работает для длин волн фотона, меньших, чем дебройлевская длина волны электрона (масштаб пикометры). Поскольку ничего электрически заряженного легче электрона в природе пока не обнаружено, то и Комптон-эффекта для более коротковолновых фотонов не наблюдено. Ну и что? Рентгеновские и гамма-фооны таки длину волны меняют. Уже 85 лет это известно и наблюдаемо. И стало быть, она у них есть. А стало быть, ваше утверждение о том, что фотон длину волны менять не может, ошибочно. И длину волны меняет с дорогой душой, и энергию.

Вероятно, при взаимодействии фотона с электроном происходит перераспределение энергии между фотоном и электроном.

Нет, это не "вероятно", это совершенно точно.

Но первичный фотон уничтожается, а потом уже излучается другой фотон.

Простите, но это очень вольная трактовка процесса. В силу принципа неопределенности и принципа тождественности подобные утверждения применимы почти к любым процессам с участием элементарных частиц. Но это спокойно уживается с утверждением, что данная частица (в данном случае фотон) таки меняет СВОЕ состояние (энергию, импульс, ориентацию спина, и т.д. и т.п.). Подробности - в любом популярном изложении основ квантовой механики. В любом случае это Ваше утверждение никак не опровергает возможности изменения длины волны фотона.

И - еще раз - фотон обладает ярко выраженными дифракционными и поляризационными свойствами и его поведение описывается формализмом, сожержащим длину волны в виде параметра. Поробуйте расчитать, например, необходимое качество поверхности зеркала в рефлекторе, выкинув из формул длину волны - хрен у Вас что получится. Поэтому понятие "длина волны фотона" вполне имеет право на жизнь и имеет ясный физический смысл, что бы Вы не утверждали.

Электроны в ЭЛТ(CRT) движутся в узком пучке. И при этом особо
не стремятся разлететься веером(конусом) из-за немалых сил
электростатического отталкивания.

У меня сил и времени не хватит подробно отвечать на все Ваши вскользь брошенные рассуждения. Однако об электронных пучках, их фокусировке и их расхождении за счет кулоновских сил книг и материалов в тырнете вагон и маленькая тележка. Если Вас это реально интересует - возьмите и прочитайте.

igor_da_bari
17.11.2009, 13:52
Конечно, под словом «увидеть» подразумевал более широкий смысл; не только визуальные наблюдения, но и с помощью различных приборов.
Но давайте снова представим наблюдателя в двухмерном пространстве. Находясь на поверхности сферы, он может наблюдать, как расширяется его вселенная, гравитационное воздействие некой огромной массы, но установить, что в этот момент происходит закачка газа в сферу, он не сможет. Разве это не похоже на то, что мы наблюдаем в своей вселенной?

Сергей, при чем тут закачка газа? Ну нельзя ж аналогию с шариком продолжать вплоть до деталей. Ну возьмите вместа шарика сверхпрочную пленку, натяните ее на разлетающиеся осколки взорвавшейся гранаты - никакой закачки не будет тогда. Мы не обсуждаем причину и механизм расширения. Мы обсуждаем геометрическую интерпретацию и следствия. Если говорить о механизме - это будет уже совсем другой разговор.

igor_da_bari
17.11.2009, 14:04
Мдаа... А ведь так и умрем с такими вот вопросами. Обидно :(.

Двадцать лет прошло, но ничего не изменилось, все те же вопросы, все те же рассуждения.

Юрий, научно-популярные сайты в интернете переполнены информацией по этим вопросам (в том числе и касающейся прогресса последних лет). Например, сайт http://elementy.ru/. Выходят (пусть и малыми тиражами) российские и переводные научно-популярные книжки на эту тему. Существуют журналы Science, Nature, Наука и жизнь, Химия и жизнь и т.д.

Если двадцать лет на все это не обращать внимания, то естественно, за 20 лет ничего не изменится. Если 50 лет все это игнорировать, то и за 50 лет ничего не произойдет, уверяю Вас. И вопросы будут те же самые, и рассуждения.

Но если хотя бы раз в несколько месяцев хотя бы часть всего вышеперечисленного хотя бы по диагонали проглядывать, то появятся ответы на вопросы, и рассуждения тоже будут непрерывно менятся в соответствии с реальным прогрессом в данной области человеческого знания.

Вот, навскидку: http://elementy.ru/lib/430484, http://elementy.ru/lib/25560?context=2455814

Ar-Gen-Tum
17.11.2009, 15:06
...
И - еще раз - фотон обладает ярко выраженными дифракционными и поляризационными свойствами и его поведение описывается формализмом, сожержащим длину волны в виде параметра. Поробуйте расчитать, например, необходимое качество поверхности зеркала в рефлекторе, выкинув из формул длину волны - хрен у Вас что получится. Поэтому понятие "длина волны фотона" вполне имеет право на жизнь и имеет ясный физический смысл, что бы Вы не утверждали.
...

Работал у нас конструктор. Интересный товарисч. Много всякого рассказывал.
Вот шарикоподшипниковый завод. Шарики прут по конвейеру.
Они не все одинаковые и некоторые не лезут в допуска по геометрии.
Как рассортировать шарики?
В свое время один человек предложил замечательную идею.
Шарики движутся по конвейеру и по одному падают на металлическую плиту
и отскакивают от нее. Отскакивают в зависимости от своей формы.
Каждый попадает в свое место.
Таим образом одинаковые шарики отскакивают в одном направлении
и под одним углом.
Если поверхность на которую они падают будет иметь шероховатость
соизмеримую с размером шарика, то шарики будут рассеиваться.
Один попадет на впадинку, а другой на горбик, а третий в какую нить
промежуточную точку.
Как видно вовсе не обязательно при этом наделять шарик длиной волны.
Однажды в армии произвели выстрел из АК калибра 5.45 в ствол ели.
Видно было, как пуля на выходе резко ушла в сторону от первоначального направления.
Оно и понятно - переход из одной среды в другую. Т.е. эффект преломления.
И опять не обязательно наделять пулю длиной волны.
Де-Бройлевская длина волны в случае шарика и пули весьма и весьма мала.

igor_da_bari
17.11.2009, 15:15
Работал у нас конструктор. Интересный товарисч. Много всякого рассказывал.
Вот шарикоподшипниковый завод. Шарики прут по конвейеру.
Они не все одинаковые и некоторые не лезут в допуска по геометрии.
Как рассортировать шарики?
В свое время один человек предложил замечательную идею.
Шарики движутся по конвейеру и по одному падают на металлическую плиту
и отскакивают от нее. Отскакивают в зависимости от своей формы.
Каждый попадает в свое место.
Таим образом одинаковые шарики отскакивают в одном направлении
и под одним углом.
Если поверхность на которую они падают будет иметь шероховатость
соизмеримую с размером шарика, то шарики будут рассеиваться.
Один попадет на впадинку, а другой на горбик, а третий в какую нить
промежуточную точку.
Как видно вовсе не обязательно при этом наделять шарик длиной волны.
Однажды в армии произвели выстрел из АК калибра 5.45 в ствол ели.
Видно было, как пуля на выходе резко ушла в сторону от первоначального направления.
Оно и понятно - переход из одной среды в другую. Т.е. эффект преломления.
И опять не обязательно наделять пулю длиной волны.
Де-Бройлевская длина волны в случае шарика и пули весьма и весьма мала.

Насчет шарикоподшипников врать не буду. Не разбираюсь. Но мы с Вами о фотонах вроде говорили...

M.Sergey
17.11.2009, 18:00
Сергей, при чем тут закачка газа? Ну нельзя ж аналогию с шариком продолжать вплоть до деталей.
Конечно, я привожу аналогию с шариком, чтобы легче было представить возможность существования других измерений. Ну не возможно мне вообразить «темную материю», которая имеет огромную массу и при этом ни как не взаимодействует с видимыми веществами и излучениями, только гравитационно! Но если представить, что наша трехмерная вселенная является поверхностью четырехмерной – сразу все становится на свои места.

igor_da_bari
17.11.2009, 18:34
Ну не возможно мне вообразить «темную материю», которая имеет огромную массу и при этом ни как не взаимодействует с видимыми веществами и излучениями, только гравитационно!

Так ведь всем это трудно представить. И именнно поэтому так долго и упорно идут поиски каких-то других проявлений dark matter. С утверждением в качестве базисной (хотя строго говоря, еще не подтвержденной экспериментально) концепции "темной энергии" для проблемы "темной материи" ничего не изменилось. 25-30% гравитационно проявляющего себя "нечты" в любом случае не темная энергия, а в какой-то форме вещество. И поиски этой нечты продолжаются.

С другой стороны не могу не заметить, что мотив "не могу вообразить" уже более ста лет как не работает в качестве критерия отбора приемлемых физических гипотез. Никто из людей не может вообразить ни дифракцию электронов, ни принцип неопределенности, ни вообще квантовомеханических свойств материи. Рацональная природа наделила нас способностью вообразить только то, что может происходить в масштабах расстояний, стравнимым с размером наших организьмов и в масштабах времени, сравнимых с теми, что мы можем отличить от нуля (а это никак не меньше, чем несколько десятых секунды. Ни Вы, ни я и никто из людей не в сотстоянии вообразить себе интервал 0.001 секунды (который "с точки зрения" ядерных объектов трудно отличим от бесконечности). Мы можем реально представить себе 10-метровой высоты столб. Но не можем представит себе реальный размер Земли или, скажем, расстояния до Солнца. Равно как и размер атомного ядра (фемтометр примерно). То, что происходит на временных и пространственных масштабах, отличных от представимых нами, мы можем только описать формулами, но не представить наглядно. А происходят там вещи, которые радикально отличаются от тех, что происходят на наших масштабах... И которые сплошь и рядом вообразить невозможно. Тем не менее они вполне реальны. Это доказывается тем, что мы умеем обнаруживать на нашей шкале времен и расстояний проявления этих процессов. Причем, заметьте, ПРЕДСКАЗАННЫЕ проявления. И не просто предсказанные, а КОЛИЧЕСТВЕННО предсказанные.

Воображению поддается то, что происходит на пространственных масштабах примерно от 0.0001 метра до 10000 метров. 8 порядков величины. Размеры реальных объектов, с которыми приходится иметь дело могут быть на 10 и более порядков меньше нашей нижней границы. А могут быть на 22 (!!!) порядка выше верхней границы. То есть непосредственно нашим чувствам, мыслям, воображению и интуиции доступно 8 порядков из более чем 40. И - 8-9 порядков из 40 на временной шкале. То есть большая часть процессов проходит совсем не в наших масштабах. И по законам, которые в нашей шкале без специального и очень дорогого оборудования вообще никак себя не проявляют. Нет ничего удивительного в том, что вообразить мы их никак не можем :)

Но если представить, что наша трехмерная вселенная является поверхностью четырехмерной – сразу все становится на свои места.

Вы не сможете РЕАЛЬНО представить себе трехмерную поверхность, по причинам изложенным выше. И никто не сможет :) Поэтому вряд ли что-то встанет на свои места в том смысле, что Вы сможете вообразить. ;)

M.Sergey
17.11.2009, 19:05
Так ведь всем это трудно представить. И именнно поэтому так долго и упорно идут поиски каких-то других проявлений dark matter. С утверждением в качестве базисной (хотя строго говоря, еще не подтвержденной экспериментально) концепции "темной энергии" для проблемы "темной материи" ничего не изменилось. 25-30% гравитационно проявляющего себя "нечты" в любом случае не темная энергия, а в какой-то форме вещество. И поиски этой нечты продолжаются.
Ну-ну, посмотрим, что они найдут.:cool:
Вы не сможете РЕАЛЬНО представить себе трехмерную поверхность, по причинам изложенным выше. И никто не сможет :) Поэтому вряд ли что-то встанет на свои места в том смысле, что Вы сможете вообразить. ;)
Представить невозможно, предположить вполне.;)

igor_da_bari
17.11.2009, 19:16
Ну-ну, посмотрим, что они найдут.:cool:

Обязательно посмотрим. "Подумаешь, гравицап - и не такое доставали" (с) :) Истории такого рода в физике занимают десятилетия и чаще всего заканчиваются совсем не тем, с чего начинались. Задуманная в 50-е годы Дэвисом проверка стандартной солнечной модели привела к тому, о чем тогда, 60 лет назад, никто и думать не думал - к обнаружению осцилляций нейтрино. Лет через 30 после того, как начала развиваться электрослабая теория, в ЦЕРНЕ Рубиа впервые увидел W и Z бозоны с потрясающим совпадением всех свойств с предсказанными теорией в пределах пары процентов. И история с темной материей непременно закончится чем-нибудь интересным и, скорее всего, неожиданным.

Представить невозможно, предположить вполне.;)

Ну так вот я о том и говорю. Что невозможность представить - не критерий. Большую часть того, что в этом мире происходит, никто из нас представить не в состоянии. И в этом нет беды, если можно предположить, описать и обнаружить.

С темной материей именно это пока и происходит - представить не можем, но не заметить не можем тоже. Факты - вопиют. А стало быть есть надежда, что удастся и обнаружить и описать :)

Юрий
17.11.2009, 22:54
Юрий, научно-популярные сайты в интернете переполнены информацией по этим вопросам (в том числе и касающейся прогресса последних лет). Например, сайт http://elementy.ru/. Уфф, замучился читать, уже ум за разум заходит :).

Nekkar
18.11.2009, 00:39
Кесаря Борис транклюкировал нахрен. O:)

Как-то это уж слишком обыденно. Рогатка это все же менее тривиальный вариант. Жаль...

Ar-Gen-Tum
18.11.2009, 02:12
...
Но мы с Вами о фотонах вроде говорили...
Может и не совсем по теме, но фотон очень уж важная деталь в исследованиях по теме.
Очень интересная и длинная у него история. Такой пожалуй нет ни у какой другой частицы.
Фотон -Википедия (http://ru.wikipedia.org/wiki/%D0%A4%D0%BE%D1%82%D0%BE%D0%BD)
В большинстве теорий, разработанных до XVIII века, свет рассматривался как поток частиц. Одна из первых таких теорий была изложена в «Книге об оптике» Ибн ал-Хайсамом в 1021 году. В ней ученый представлял световой луч в виде потока мельчайших частиц, которые «испытывают нехватку всех заметных качеств, кроме энергии».
Офигеть, почти 1000 лет назад. Без приборов, чисто на научной интуиции.
Философия однако - "любовь к мудрости".
Наше время:

...
Некоторые физики[42] продолжали разрабатывать полуклассические модели, в которых электромагнитное излучение не считалось квантованным, но вопрос получил свое разрешение только в рамках квантовой механики. Идея фотонов при объяснении физических и химических экспериментов стала общепринятой к 70-м годам XX в. Все полуклассические теории были окончательно опровергнуты в 70-х и 80-х годах в экспериментах по фотонной корреляции[43]. Таким образом, идея Планка о квантовых свойствах электромагнитного излучения и развитая на её основе гипотеза Эйнштейна считаются доказанными.

Breath
18.11.2009, 04:41
Если Вселенная бесконечна , то каждая её точка может являться центром .

ZamaZzZka
18.11.2009, 10:12
Ну-ну, посмотрим, что они найдут.

А какая у вас теория? Что удерживает структуру галактики?

M.Sergey
18.11.2009, 12:12
А какая у вас теория? Что удерживает структуру галактики?
То, что принято называть «темной материей», то и удерживает. Вот только я сомневаюсь, что эта материя находится в нашем трехмерном пространстве.

Weirdie
19.11.2009, 00:38
Честно говоря, не представляю, как нечто материальное может находиться в одном измерении и не находиться при этом во всех остальных.

АлександрВЛ
19.11.2009, 10:35
Работал у нас конструктор. Интересный товарисч. Много всякого рассказывал.
Вот шарикоподшипниковый завод. Шарики прут по конвейеру.
Они не все одинаковые и некоторые не лезут в допуска по геометрии.
Как рассортировать шарики?
В свое время один человек предложил замечательную идею.
Шарики движутся по конвейеру и по одному падают на металлическую плиту
и отскакивают от нее. Отскакивают в зависимости от своей формы.
Каждый попадает в свое место.
Таим образом одинаковые шарики отскакивают в одном направлении
и под одним углом.
Если поверхность на которую они падают будет иметь шероховатость
соизмеримую с размером шарика, то шарики будут рассеиваться.
Один попадет на впадинку, а другой на горбик, а третий в какую нить
промежуточную точку.
Как видно вовсе не обязательно при этом наделять шарик длиной волны.
.
Это ж какого диаметра должен быть шарик, что бы реагировать на шероховатости плиты, ну и как следствие сам шарикоподшипник? Не смешите. Умный человек такого предложить не мог. особенно с фразой "шарики прут по конвейру".

Ar-Gen-Tum
19.11.2009, 10:42
Это ж какого диаметра должен быть шарик, что бы реагировать на шероховатости плиты. Не смешите. Умный человек такого предложить не мог.
Возможно в вашем понимании шероховатость - это от долей до десятков микрон.
А в широком смысле может быть абсолютно любой.
К примеру шероховатость земной поверхности может быть от метров до километров.
:)
Если Вы квотите сообщение, то перечитывайте его до тех пор пока до Вас
не дойдет смысл написанного.

АлександрВЛ
19.11.2009, 12:35
Я то все понимаю, но не могу связать воедино, как конструктор!!!!!! работающий на заводе, производящем шарикоподшипники!!!! умудрился высказать идею о проверке допусков шариков таким образом!!! Я к данному вопросу подхожу конкретно. Сам работаю в металлообрабатывающей сфере. Знаю что это такое. Поэтому и в голове не укладывается. Не надо так говорить про конструкторов. Это обидно, знаете ли.

Andre_aka_Lilit
19.11.2009, 14:05
Я то все понимаю, но не могу связать воедино, как конструктор!!!!!! работающий на заводе, производящем шарикоподшипники!!!! умудрился высказать идею о проверке допусков шариков таким образом!!! Я к данному вопросу подхожу конкретно. Сам работаю в металлообрабатывающей сфере. Знаю что это такое. Поэтому и в голове не укладывается. Не надо так говорить про конструкторов. Это обидно, знаете ли.

Ну Вы как имеющий отношение к металлообработке должны знать о т.н. классах чистоты.;)

megabobr
19.11.2009, 14:16
Среди наиболее поразительных, неожиданных и важных достижений прошлого столетия (или даже более) находится открытие тёмной материи и тёмной энергии, называемых общим термином "тёмная область" ("dark sector"). Около колоссальных 96% всей Вселенной принадлежит тёмной области. Тёмная материя невидима в том смысле, что она не излучает фиксируемых форм энергии (света, радиоволн…), но определённо существует, потому что её гравитация оказывает поддающийся измерению эффект на звёзды – объекты, которые можно наблюдать. Среди всей материи космоса 90% - это тёмная материя, в то время как галактики и светила являются незначительными "дополнениями". Пока науке неизвестно, что из себя представляет тёмная материя. По одной из версий, она состоит из элементарных частиц, отличных от входящих в атомы.
С другой стороны, тёмная энергия – это не форма материи вообще (и не буквально "тёмная" – это просто метафора, передающая её загадочность). Она является источником силы, ускоряющей расширение Вселенной, и, если основываться на текущих знаниях о физике элементарных частиц, происходит из вакуума, который имеет квантовые свойства, свидетельствующие о передаче энергии в космос. Альтернативная точка зрения заключается в следующем: тёмная энергия – свойство гравитации, которое обуславливает появление отталкивающих сил огромных масштабов.
Интересно как она образовалась ? И вобще это одна из самых загадочных вещей во Вселенной !!!!

АлександрВЛ
19.11.2009, 14:29
Знаю я об этом. Так же знаю о пластичности, эластичности, текучести и прочих ости... и ести. У меня не укладывается в голове КАК КОНСТРУКТОР МОГ ПРЕДЛОЖИТЬ ИЗМЕРЕНИЕ РЕАЛЬНО ВЫПУСКАЕМЫХ ПОДШИПНИКОВ ТАКИМ МЕТОДОМ? Бред. И вопрос: видели ли ли Вы под микроскопом, что из себя представляет 12 класс или по другому, грубо говоря, треть микрона? Поэтому рискну предположить, что эту байку Вы придумали сами. Если это так-ничего страшного. Просто на досуге взгляните в электронный микроскоп. Увидите много интересного. Да и на поверхность шарика взгляните. Просто если говорить простым языком, то для обозначенных Вами размеров шарика, подойдет наверное такое определение: сбрасываем ежика, свернувшегося клубком, на густой ельник и смотрим куда он отскочит....O:)

megabobr
19.11.2009, 15:17
Может и не совсем по теме, но фотон очень уж важная деталь в исследованиях по теме.
Очень интересная и длинная у него история. Такой пожалуй нет ни у какой другой частицы.
Фотон -Википедия (http://ru.wikipedia.org/wiki/%D0%A4%D0%BE%D1%82%D0%BE%D0%BD)

Офигеть, почти 1000 лет назад. Без приборов, чисто на научной интуиции.
Философия однако - "любовь к мудрости".
Наше время:

я думаю учоные того времени не имея приборов, точных сведений и данных просто напросто умели воображать . Чего нам сейчас нехватает !!!O:) O:) O:)

Andre_aka_Lilit
19.11.2009, 15:49
Знаю я об этом. Так же знаю о пластичности, эластичности, текучести и прочих ости... и ести. У меня не укладывается в голове КАК КОНСТРУКТОР МОГ ПРЕДЛОЖИТЬ ИЗМЕРЕНИЕ РЕАЛЬНО ВЫПУСКАЕМЫХ ПОДШИПНИКОВ ТАКИМ МЕТОДОМ? Бред. И вопрос: видели ли ли Вы под микроскопом, что из себя представляет 12 класс или по другому, грубо говоря, треть микрона? Поэтому рискну предположить, что эту байку Вы придумали сами. Если это так-ничего страшного. Просто на досуге взгляните в электронный микроскоп. Увидите много интересного. Да и на поверхность шарика взгляните. Просто если говорить простым языком, то для обозначенных Вами размеров шарика, подойдет наверное такое определение: сбрасываем ежика, свернувшегося клубком, на густой ельник и смотрим куда он отскочит....O:)


Что я придумал? Классы чистоты поверхностей?:confused:
Кроме того имею 5 рабочий разряд по специальности "Наладка и эксплуатация станков с ЧПУ" и техническое образование по этой же специальности, но это было в прошлой еще жизни ;)

Ar-Gen-Tum
19.11.2009, 19:51
Знаю я об этом. Так же знаю о пластичности, эластичности, текучести и прочих ости... и ести. У меня не укладывается в голове КАК КОНСТРУКТОР МОГ ПРЕДЛОЖИТЬ ИЗМЕРЕНИЕ РЕАЛЬНО ВЫПУСКАЕМЫХ ПОДШИПНИКОВ ТАКИМ МЕТОДОМ? Бред.
...

Написал, что слышал.
Посмотрел на медни в инете инфу по производству шариков для подшипников.
Процесс весьма интересный.
Расписаны основные этапы, но о контроле не написано.
Возможно, что массовый контроль и нет смысла производить.
Возможно, что достаточно выборочного. Штуку из миллиона. :)
Но не думаю, что каждый промеряют микрометром и контролируют
поверхность под микроскопом. :)
Раз Вы специалист, так выскажите идею, как проверить форму шариков
на потоке в 100 штук в секунду. Ну хотя-бы одну штуку в секунду. :)
Под формой понимается общая геометрия шарика.
Даже если он будет идеально гладкий, то есть-ли 100% гарантия,
что он идеально сферический.
И если задать направление, то будет-ли разница при отражении(отскоке)
шарика со сферической поверхностью и с несферической?
А вобще-то разговор был о фотонах. Повторяться не буду. Желающие могут отлистать назад.

КентаVR
19.11.2009, 20:29
Нет, уж лучше о подшипниках. Вы уже всех заинтриговали :D

thirtyseconds
20.11.2009, 01:25
На параллельном форуме идет дискуссия на тему красного смещения. Очень содержательный пост - http://www.astronomy.ru/forum/index.php/topic,57375.msg1087946.html#msg1087946

АлександрВЛ
20.11.2009, 07:57
Написал, что слышал.
Посмотрел на медни в инете инфу по производству шариков для подшипников.
Процесс весьма интересный.
Расписаны основные этапы, но о контроле не написано.
Возможно, что массовый контроль и нет смысла производить.
Возможно, что достаточно выборочного. Штуку из миллиона. :)
Но не думаю, что каждый промеряют микрометром и контролируют
поверхность под микроскопом. :)
Раз Вы специалист, так выскажите идею, как проверить форму шариков
на потоке в 100 штук в секунду. Ну хотя-бы одну штуку в секунду. :)
Под формой понимается общая геометрия шарика.
Даже если он будет идеально гладкий, то есть-ли 100% гарантия,
что он идеально сферический.
И если задать направление, то будет-ли разница при отражении(отскоке)
шарика со сферической поверхностью и с несферической?
А вобще-то разговор был о фотонах. Повторяться не буду. Желающие могут отлистать назад.
1.Я не специалист по проверке формы шариков. Это Вам в отдел технического контроля на шарикоподшипниковом заводе.
2. Проведите опыт и ответьте на поставленный вопрос. Это лучше чем сострясать воздух. Тем более опыт несложный.
3. Про шарики говорить хватит. Это безсодержательно.

igor_da_bari
20.11.2009, 13:10
А вобще-то разговор был о фотонах.

О! Наконец-то Вы вспомнили, Геннадий. Что разговор был о фотонах, о которых Вы утверждали, что они не могут иметь длины волны. А будучи припертым к стенке на эту тему, неожиданно переключились на шарики. Теперь Вас с шариками приперли - Вы, стал быть, назад к фотонам зовете? :)

А ну как там ход беседы потребует от Вас какого-нибудь содержательного предметного высказывания по существу? Опять про шарики начнете, что ли?

Нет, Геннадий, Вы как хотите, но я из этой беседы выхожу. У меня вестибулярный аппарат слабый и от этой карусели в глазах мелькание, сердце ёкает и блевать тянет. Тем более толку от переливания из пустого в порожнее в столь свободном и ни к чему не обязывающем стиле я все равно никакого не вижу...

Ar-Gen-Tum
20.11.2009, 18:08
О! Наконец-то Вы вспомнили, Геннадий. Что разговор был о фотонах, о которых Вы утверждали, что они не могут иметь длины волны. А будучи припертым к стенке на эту тему, неожиданно переключились на шарики. Теперь Вас с шариками приперли - Вы, стал быть, назад к фотонам зовете? :)

А ну как там ход беседы потребует от Вас какого-нибудь содержательного предметного высказывания по существу? Опять про шарики начнете, что ли?
...
В языковых упражнениях всегда впереди будет тот у кого этот аппарат
лучше развит. :)
Я ни с кем не собираюсь состязаться. :)

megabobr
20.11.2009, 18:39
Кто бы мог подумать что тайну гравитации инерции и движения не надо далеко искать. В тёплый летний день леч на зелёную траву и взглянуть глазами и умом на голубое голубое небо и увидеть, что всем в мире правят фотоны. Те самые, что нам солнце бесплатно каждый день ливнем разливает по земле. Не много ума надо, что б понять, что без солнечных фотонов жизни не было б. А философы говорят, что жизнь - движение. Они уже знали, что тайна движения в фотонах. Для философов вроде это и не тайна. Но они громко не кричали, что б их услышал великий Эйнштейн, он бы пространство не кривил и теорию гравитации создал бы фотонную. Так что же это за зверь фотон? В начале 20века Эйнштейн понял, что фотон волна и частица. Признаки волны, когда у фотона малая энергия. Когда у фотона большая энергия, длина волны уменьшается и у фотона проявляются признаки частицы. Эйнштейн понял, что и фотон и частица это всё энергия. У него есть знаменитая формула E=mc кв. У фотона энергия перемещается по прямой в пространстве, то есть перемещается сам фотон. А вот у частицы энергия перемещается в пространстве по кругу. В общем энергия в частице как белка в колесе. Для нас людей частица не движется, мы её движение по кругу не ощущаем. Это мёртвая энергия. Для нас она не существует. Это как эйнштейновские чёрные дыры. Вот из этой мёртвой материи и мы состоим. Но из повседневного опыта мы знаем и философы знают, что всё движется, всё именяется. Вот тут то и мысль, а не фотоны ли движением заведуют. Да фотоны. Это версия, а доказательство........
Логическое доказательство фотонной теории движения всего сущего.
Есть наука термодинамика, которая доказывает, что теплота это внутреннее движение микрочастиц в макротеле. Сильнее внутреннее движение, больше теплоты. При горении выделяется тепло, огонь изменяет свойства материи. Создателей термодинамики изменение свойств материи и интересовало.:roll: :roll:

igor_da_bari
20.11.2009, 18:42
Кто бы мог подумать что тайну гравитации инерции и движения не надо далеко искать. В тёплый летний день леч на зелёную траву и взглянуть глазами и умом на голубое голубое небо и увидеть, что всем в мире правят фотоны. Те самые, что нам солнце бесплатно каждый день ливнем разливает по земле. Не много ума надо, что б понять, что без солнечных фотонов жизни не было б. А философы говорят, что жизнь - движение. Они уже знали, что тайна движения в фотонах. Для философов вроде это и не тайна. Но они громко не кричали, что б их услышал великий Эйнштейн, он бы пространство не кривил и теорию гравитации создал бы фотонную. Так что же это за зверь фотон? В начале 20века Эйнштейн понял, что фотон волна и частица. Признаки волны, когда у фотона малая энергия. Когда у фотона большая энергия, длина волны уменьшается и у фотона проявляются признаки частицы. Эйнштейн понял, что и фотон и частица это всё энергия. У него есть знаменитая формула E=mc кв. У фотона энергия перемещается по прямой в пространстве, то есть перемещается сам фотон. А вот у частицы энергия перемещается в пространстве по кругу. В общем энергия в частице как белка в колесе. Для нас людей частица не движется, мы её движение по кругу не ощущаем. Это мёртвая энергия. Для нас она не существует. Это как эйнштейновские чёрные дыры. Вот из этой мёртвой материи и мы состоим. Но из повседневного опыта мы знаем и философы знают, что всё движется, всё именяется. Вот тут то и мысль, а не фотоны ли движением заведуют. Да фотоны. Это версия, а доказательство........
Логическое доказательство фотонной теории движения всего сущего.
Есть наука термодинамика, которая доказывает, что теплота это внутреннее движение микрочастиц в макротеле. Сильнее внутреннее движение, больше теплоты. При горении выделяется тепло, огонь изменяет свойства материи. Создателей термодинамики изменение свойств материи и интересовало.:roll: :roll:

Я обрыдался читаючи... :D

megabobr
20.11.2009, 18:55
Извините если я чушь несу !!!!! просто меня занесло!!! Хи-Хи](*,) ](*,) :(

Sinope
18.01.2010, 17:09
Потому что:
Например наличие реликтового излучения, плотность и температуру которого заранее и очень точно предсказал Гамов исходя из модели "Большого Взрыва" . Или наблюдаемая крупномасштабная структура Вселенной, которая при численном моделировании, включающем расширение, именно такой и должна получаться.

Все-таки хотелось бы подробнее. Кем и когда оно производилось (можно без матриц и алгоритмов). Если Вы о нем упомянули, то должны быть в курсе, очень надеюсь.
Или хим. состав Вселенной - он тоже зависит от того, каким было начало и как все развивалось. Он тоже с расширением "дружит". Соотношение наблюдаемого количества барионов и лептонов.

Тут тоже заковыка. Я имею в виду барионную асимметрию.
------------
Но в целом, я тоже большой поклонник ТБВ.:)

Влад
18.01.2010, 17:44
Но в целом, я тоже большой поклонник ТБВ.
Земляка приветствую. Ждём дальнейших сообщений.

Sinope
23.01.2010, 09:40
Владbvbh
Я к чему веду: оптимизм уважаемого igor_da_bari мне понятен; Вселенная достаточно хорошо описывается в больших масштабах в ОТО, и локально - в КМ, однако гравитация ни там ни сям ни к месту - энергия гравитационного поля не локализуется. Оно квантуется только при некоторых искусственных предположениях. Ключевое из них - отсутствие масс (!). Без этого нельзя ввести такую частицу как гравитон. И любой вариант объединённой квантовой теории получается неперенормируемым - не пригоден для расчётов. Поэтому любая квантовая теория, включающая гравитацию, получается ни к чёрту не годной.
:(
В общем, пока проблем достаточно.

c4ever
01.06.2010, 13:01
Очень интересная тема!
Извините, что вот так врываюсь, но позволю себе высказать несколько своих соображений. Попрошу ногами больно не бить, т. к. в высшей физике я мало что понимаю. Именно понимаю, т. к. знать и понимать - это две совершенно разные вещи.

По поводу межгалактических "пустот", неравномерном распределении обычного вещества и образовании галактик: в последние годы набирает обороты теория, подтвержденная некоторыми практическими наблюдениями, о том, что в центре Млечного Пути и подавляющего большинства (если не всех) галактик находится сверхмассивная черная дыра. Исходя из этого предположения можно привести сразу несколько новых предположений (так, как это видится мне): на заре образования вселенной, когда гравитационные силы локально начали преобладать над излучением, когда пошла пресловутая "рекомбинация" (которая с теоретической точки зрения все-таки ограничивает наше поле зрения, т. к. до нее фотоны не могли свободно перемещаться), начали образовыватсья первые протозвезды огромных размеров и масс, "выгоревшие" за короткие сроки, впоследствии сколлапсировавшие в черные дыры. Вокруг этих "протодыр" началась "пляска" оставшегося вещества. Со временем все это разлеталось, или, если хотите, расширялось во все стороны, и каждая такая дыра стала центром галактики. В постах на первых страницах кто-то выдвигал предположения, о том что обычная материя, из которой состоят галактики, выбрасывается из их центров. Мне видится совершенно обратное: вещество "падает" на черную дыру, как вода закручивается в водосток в раковине. У меня почему-то напрашивается именно такая аналогия, особенно при виде спиральной галактики. Выбросы вещества из центров активных галактик перпендикулярно их плоскости не что иное, как действие на ионизированное вещество мощнейщего магнитного поля, создаваемого веществом, вращающимся вокруг черной дыры с релятивистскими скоростями. Вышеупомянутые протозвезды, из которых образовались сверхмассивные черные дыры, сейчас мы наблюдаем как всем известные квазары...

Основной же причиной, по которой я нашел эту тему в нете, была мысль возникшая у меня вчера при прочтении статьи (http://ru.wikipedia.org/wiki/%D0%A0%D0%B5%D0%BB%D0%B8%D0%BA%D1%82%D0%BE%D0%B2%D 0%BE%D0%B5_%D0%B8%D0%B7%D0%BB%D1%83%D1%87%D0%B5%D0 %BD%D0%B8%D0%B5) в вики о современных данных о реликтовом излучении. Эти данные пошатнули чуть-ли не постулат об однородности и изотропности вселенной. Особенно это касается дипольной составляющей анизотропии реликтового излучения. Вдумайтесь в интерпретацию этого явления, как "движение относительно излучения".. Я не очень хорошо знаком с ОТО, но мне кажется, что даже в ее рамках такая формулировка неприемлема. Возможно движение только относительно источника излучения. Вот тут-то и начинается самое интересное.. Если предположить, что реликтовое излучение существует с той самой поры рекомбинации, то источником реликтового излучения являются "стенки" раздувающегося "шара времени", и если дипольная составляющая неоднородности реликтового излучения вызвана именно тем, одна из этих "стенок" отдаляется от нас быстрее, чем другая... вы уже наверное догадались к чему я клоню. Проведя прямую вдоль полюсов этого реликтового диполя мы попадем прямо в центр вселенной!! :)) Только вот способ определить расстояние до него у меня пока еще не придумался)
И еще.. может быть это самое реликтовое излучение и есть "жестоко красносмещенное" излучение рекомбинирующегося вещества...
Может, конечно, мои рассуждения здешним "волкам" покажутся почти школьными, но в какой-то момент мне показалось, что меня осенило :))
Было у меня еще много всяческих соображений по этому поводу, в частности мне до сих пор не дает покоя двойственность эффекта Доплера, и еще очень многие вещи, но сейчас в голове все слегка перепуталось, может потом напишу)

Игорь_
17.07.2010, 18:14
Что есть центр Вселенной?
Наверно есть ведь когдато от куда-то большоё взрыв начался :D

Сашка
19.07.2010, 20:25
Здраствуйте! Читал в одной статье местная группа галактик и скопление в Деве обращаются вокруг одного центра называется он вроде бы "Автарктор" не ручаюсь за точность. Сатья очень интересная! Вспомню где читал, сообщу.

thirtyseconds
19.07.2010, 21:26
Аттрактор. От английского attract. Притягивать, привлекать.

http://ru.wikipedia.org/wiki/Великий_аттрактор

igor_da_bari
19.07.2010, 22:10
Здраствуйте! Читал в одной статье местная группа галактик и скопление в Деве обращаются вокруг одного центра называется он вроде бы "Автарктор" не ручаюсь за точность. Сатья очень интересная! Вспомню где читал, сообщу.

Бум ждать, когда вспомните. А если не вспомните, вспомним сами школьную программу. Так что не переживайте сильно. Если у вас память не восстановится, то уровень образования в России не слишком упадет. Спите спокойно.

thirtyseconds
19.07.2010, 22:41
Игорь, не травмируйте окружающих :) В конце концов в школе сейчас в лучшем случае про галактики. Про аттракторы там только в рамках того самого. Английского.

igor_da_bari
19.07.2010, 22:48
Игорь, не травмируйте окружающих :) В конце концов в школе сейчас в лучшем случае про галактики. Про аттракторы там только в рамках того самого. Английского.

Spriechst Du Deutsch? :D

thirtyseconds
19.07.2010, 22:56
Нет, хотя это и безобразие, поскольку долбили в меня этот язык очень долго. Это даже немного стыдно. Но в принципе если месяц почитать то можно вспомнить. Собственно я даже собираюсь, но щас время нет.

Sly
19.07.2010, 23:52
Судя по статье с Вики этот аттрактор так никто и не видел и судят о нем только по оказываемому им гравитационному эффекту?

igor_da_bari
20.07.2010, 04:58
Судя по статье с Вики этот аттрактор так никто и не видел и судят о нем только по оказываемому им гравитационному эффекту?

Та ни... Видят. Я помню семинар еще со своих аспирантских времен. Дядько какой-то приходил из ИКИ, кажись. Фамилию не помню. И показывал картинки с векторами скоростей для разных объектов. И там было видно такое место, куда много векторов направлено. И он его называл Great Attractor. Дело было в конце 80-х.

igor_da_bari
20.07.2010, 04:59
Нет, хотя это и безобразие, поскольку долбили в меня этот язык очень долго. Это даже немного стыдно. Но в принципе если месяц почитать то можно вспомнить. Собственно я даже собираюсь, но щас время нет.

Se parla italiano? :p

Сашка
20.07.2010, 22:49
Здраствуйте!Спасибо Трроль-Командер!"Великий Аттрактор" а статья в журнале ВПВ №2(57)2009.С.Гордиенко"Путешествие в бездну." www.wselennaya.com (http://www.wselennaya.com) хотелось ещё что нибуть почитать про Аттрактор.

Extinct
23.07.2010, 02:46
Всем здравствуйте!
Кто-нибудь все-таки может ответить где находится центр вселенной хотя бы предположительно, или хотя бы место большого взрыва?
Понятно дело что оно настолько далеко, что мы его даже не видим, понятно дело что вселенная на очень больших масштабах начинает проявлять однородность. Также мне ясно что, довольно сложно находясь внутри системы определить этот центр, особенно если не видишь даже границ системы, но это все-таки возможно.

Допустим мы находимся в произвольной точке Х.
Но при этом мы знаем.

1. Где-то большой взрыв точно был
2. Вселенная расширяется из точки взрыва.

Начинаем делать выводы.

1. Если взрыв был, то из точки взрыва началось расширение пространства
2. Раз происходило расширение, то происходило оно не моментально, а поступательно
3. Значит какие-то материально наблюдаемые объекты образовались раньше других
4. Мы можем определить какие объекты появились раньше чем другие.
5. Значит они находятся ближе к центру вселенной
6. Соответственно возле них и находится предполагаемый центр вселенной.

Допустим пространство замкнуто само на себя и мы наблюдаем два примерно одинаковых по возрасту объекта в разных концах наблюдаемого мира. Значит нужно строить 4-х мерную модель и эти объекты в данной модели будут уже находиться недалеко друг от друга и мы сможем таким образом локализовать центр вселенной по 4-х мерному пространству.

Может наша вселенная и не единственна, но это не особо важно, это уж вопрос исследований слишком отдаленного будущего. Хотя бы центр нашей вселенной бы найти, он по любому существует, потому как иначе это противоречит модели большого взрыва и расширяющийся вселенной.
Расширение в любом случае имеет вектор.

igor_da_bari
23.07.2010, 02:55
Охренеть. Пацаны вроде в Южном Бутово какую-то воронку видали. Забором огороженную. Может, оттуда?

thirtyseconds
23.07.2010, 02:55
Всем здравствуйте!
Кто-нибудь все-таки может ответить где находится центр вселенной хотя бы предположительно, или хотя бы место большого взрыва?

Все это уже обсуждалось в этой теме раз питсот...

Центра расширения нет. Это легко доказывается, поскольку в каком бы направлении мы не посмотрели, вселенная выглядит одинаково. Отсутствие центра проще осознать если перестать использовать слово "взрыв" ибо оно практически не несет, в этом случае, физического смысла.

Или можно сказать что центр везде. В качестве другой аналогии всегда приводится поверхность сферы, которая не имеет центра расширения.

Extinct
23.07.2010, 06:53
Все это уже обсуждалось в этой теме раз питсот...

Обсуждалось это не значит что пришли к правильным выводам.

Или можно сказать что центр везде. В качестве другой аналогии всегда приводится поверхность сферы, которая не имеет центра расширения.

Представь Землю.
У нее есть Северный полюс.
Представь что мы живем в двумерном мире.
Берем точку отсчета по времени условный ноль, когда вся материя была сжата в точке северного полюса.
После "взрыва" материя начала расширяться вдоль поверхности Земли, то есть как бы по сфере.
Мы, как двумерные существа, обитающие на поверхности этой сферы не можем себе представить, что Земля круглая (Гы, когда-то так оно и было :) ).
Как ты правильно сказал, мы не можем (вернее не могли раньше) осознать более высокую мерность окружающего мира, чем тот в котором живем, соответственно исходя из этих представлений мы конечно же не можем осознать где же на Земле произошел взрыв.
Но когда до наших голов начинает доходить, что мерность больше чем мы себе можем представить, и если мы перестроим модель видимой нами части вселенной под бО'льшую мерность пространства чем воспринимаем, то тогда мы уже сможем отследить, где же произошел взрыв.

Так что на счет сферы я уже давно понял, теперь ты пойми меня, модель надо 4-х мерную строить и тогда центр обнаружится. В трехмерной модели мы конечно Центра не найдем.

Ernest
23.07.2010, 09:21
Обсуждалось это не значит что пришли к правильным выводам. Резонно! ;)
Но Вы, как добросовестный исследователь вопроса, все же почитайте обсуждение сначала.

После "взрыва" материя начала расширяться вдоль поверхности Земли, то есть как бы по сфере. Похоже есть-таки смысл почитать. :)
2-мерный мир в известной всем ходячей иллюстрации расширяется не по сфере заданного радиуса начиная с какой-то точки на ее поверхности (типа все более и более кривого блина), а раздувается подобно воздушному шарику - вначале его радиус нулевой, потом все больше, больше и больше и при этом мир все время остается сферическим, только со все меньшей кривизной.

И вот с точки зрения двумерных любителей поговорить вроде нас с вами центра расширения на их сфере нет - оно и понятно он за пределами 2-мерного мира (например - в центре 3Д шара). Примерно то-же и с большим взрывом - в нашем трехмерном мире центра мы не найдем, все точки Вселенной в этом плане равноправны, надо вырваться за пределы Вселенной и посмотреть на нее со стороны, чтобы увидеть этот многомерный центр, но знание о нем едва-ли будет иметь смысл по возвращении назад в трехмерный мир. 2Д-существа вполне могут построить модель 3Д мира, по посетить или как-то использовать по другому знание о центре своей сферы они не в состоянии (не говоря уже о том, что центр расширения может и не совпадать с центром сферы - пузырь на котором они живут могли начать надувать в другом месте, то центр расширения и центр кривизны не одно и тоже).

Единственный и естественный центр в подобной ситуации связан с наблюдателем. Находясь на поверхности Земли нам кажется что мы в центре окруженные равнодалекой по всем направлениям от нас линией Горизонта. Изучая Вселенную мы видим во всех направлениях чем дальше, тем все более и более молодые объекты Вселенной на фоне горизонта в виде реликтового излучения. Но так же как и для путешественника на Земле вид горизонта примерно одинаков (хотя и зависит от местного рельефа, высоты наблюдателя на поверхностью), в какую-бы точку Вселенной не забросила судьба наблюдателя ее вид в среднем остался бы таким же.

Extinct
23.07.2010, 17:49
Резонно! ;)
Но Вы, как добросовестный исследователь вопроса, все же почитайте обсуждение сначала.

С удовольствием бы, но лень. Многа букаф.

а раздувается подобно воздушному шарику - вначале его радиус нулевой, потом все больше, больше и больше и при этом мир все время остается сферическим, только со все меньшей кривизной.

Пусть так. Далее для простоты предположим, что все-таки центр вселенной и центр "взрыва" находятся в одном и том же месте и это место - центр предполагаемой сферы, на поверхности которой находимся двумерные мы.
Далее следуя логике, двумерные мы приходим к выводу что вселенная однородна и что она так хитро устроена, что центра мы и не можем увидеть, потому как он лежит за пределами восприятия наших органов чувств.
Однако, используя математический аппарат и визуальные наблюдения, мы строим космологическую модель видимой нам части вселенной и основываясь на знаниях о ранних этапах существования вселенной, типа вот таких (http://ru.wikipedia.org/wiki/Вселенная#.D0.A2.D0.B5.D0.BE.D1.80.D0.B5.D1.82.D0. B8.D1.87.D0.B5.D1.81.D0.BA.D0.B8.D0.B5_.D0.BC.D0.B E.D0.B4.D0.B5.D0.BB.D0.B8) мы можем математически вычислить где находится условный центр вселенной. Это типа как по части поверхности сферы вычислить её кривизну, экстраполировать на другие невидимые нам участки и тем самым получить примерную модель всей вселенной, в том числе и её центра.

Единственный и естественный центр в подобной ситуации связан с наблюдателем.

Мне вот эти заморочки с ОТО не нравятся. Типа все зависит от наблюдателя. Маразм все это. Если так рассуждать Земля плоской снова станет, потому как наблюдателю так кажется. Если мы не можем увидеть начала или конца, это не значит что его нет. Нельзя принимать себя за ЦЕНТР отсчета в какой-либо системе, только потому что не можешь найти реальный центр отсчета для данной системы.

igor_da_bari
23.07.2010, 17:59
Полистав учебник по квантовой хромодинамике, он сказал: "Мне эти заморочки не нравятся. Формулы какие-то трехэтажные...".

И это было бы понятно. Ну, не нравится так не нравится. Ну, отложи эту книжку и возьми какой-нибудь там Playboy. Там все доступно и раздражения не вызывает.

Однако вместо этого человек начинает проникаться подозрениями - "раз так много непонятных лично мне букофф, значит, суки яйцеголовые, что-то скрывают от народа! (я ведь - народ)".

И начинается расследование тайн и разоблачение злоумышленников. С целью свести реальное мироздание к простой и понятной картинке из Мурзилки или Playboy.

И опять-таки все бы было ничего, если бы мироздание давало кому-то письменное обязательство быть похожим на простую и понятную всем картинку из Мурзилки.

А оно, как известно доподлинно, такого обязательства никому и никогда не давало. И не собирается.

РыбачОк
23.07.2010, 18:21
Extinct
О! И этот с ником угадал! :D

thirtyseconds
23.07.2010, 18:36
Полистав учебник по квантовой хромодинамике, он сказал: "Мне эти заморочки не нравятся. Формулы какие-то трехэтажные...".

Это он на самом деле от радости. От радости вливания в коллектив, на который, как предполагается чье-то неудовольствие ОТО должно произвести неизгладимое впечатление, о чем товарищ и не преминул второпях сообщить. Но тут все сплошь усталые и утомленные нарзаном люди :) Вот только Эрнест отличный пост выдал. И по делу.
Правда похоже впустую.

igor_da_bari
23.07.2010, 18:40
Это он на самом деле от радости. От радости вливания в коллектив, на который, как предполагается чье-то неудовольствие ОТО должно произвести неизгладимое впечатление, о чем товарищ и не преминул второпях сообщить. Но тут все сплошь усталые и утомленные нарзаном люди :) Вот только Эрнест отличный пост выдал. И по делу.
Правда похоже впустую.

Эрнест - человек бесконечного терпения и выдержки. За что я его (в числе прочего) безмерно уважаю. Но его пост и в самом деле впустую прошел. Не нужна народу правда. Народу нужно, чтобы было понятно.

Я тоже когда-то начинал с того, что пытался что-то объяснить. Но хватило меня не надолго. А если бы и хватило надолго - это бы абсолютно ничего не изменило.

thirtyseconds
25.07.2010, 10:22
Кстати это самое. Я тут ранее настаивал, что красное смещение не вызвано скоростями других галактик относительно нашей (а является следствием расширения самой вселенной). Что дескать это приводит к тому, что мы оказываемся в центре взрыва. Это неправильно. На некоей очень дальней периферии "разлета", осколки будут удаляться от друга от друга, с какого бы осколка не "посмотреть".
Но в любом случае, с какого то определенного расстояния, расширение вселенной вносит основной вклад в красное смещение.

s100p
03.10.2010, 21:31
Уф, осилить весь пост не смог, но понял, из умных утверждений, что центра вселенной не существует (хотя это не аксиома).
В таком случаи возникают вопросы - он не существовал никогда? Или с какого времени он перестал существовать? Если перестал, то почему?.
Ведь согласно теории БВ, то, что есть сейчас образовалось из точки энергии - у этой точки то в любом случаи центр был.
Еще прикалывало, не помню где читал, типа того, что в первые моменты как расширялась вселенная после БВ - получается со скорость значительно превосходящей скорость света...

igor_da_bari
04.10.2010, 00:13
Вы объясните, что такое "центр вселенной" и тогда вполне можно будет его найти. Если вы сумеете дать строгое и однозначное определение центра, то проблем с ответом не будет. А если не сумеете, тогда просто непонятно, о чем вы говорите.

Ar-Gen-Tum
04.10.2010, 00:33
Если вокруг Вселенной изобразить воображаемую сферу,
то центр этой сферы и будет центром Вселенной. :)
Воображаемая сфера может проходить через наиболее
удаленные наблюдаемые объекты.
Не исключено, что Земля и будет центром Вселенной. :)

igor_da_bari
04.10.2010, 00:38
Если вокруг воображаемой Вселенной изобразить изображаемую сферу, то найти у этой сферы воображаемый центр не составит никакого труда. Если вопрос был в этом, то я не вижу проблемы :)

s100p
24.10.2010, 22:04
Тема ни о чем! Простой обыватель, пытается выяснить, возможно ли обнаружить центр вселенной - то место где как утверждается произошел так называемый БОЛЬШОЙ ВЗРЫВ - место где находилось то микроскопическое нечто, которое начало расширяться. И что мы имеем в итоге - умные дяденьки говорят, что обыватель дурак - и этого места нет... что ж, спасибо. Мы живем не глубокой старине, и знаем, то земля не центр мира и вселенной. Но кажется, что умные дяденьки должны нам, глупым обьяснить, как понимать, что вселенная - это сфера или то, что она плоская? какова толщина этой самой сферы? налево и направо по 13,7 млр св.лет, а вверх и вниз сколько, а под углом 45 град?Помнится когда то утверждалось, что земля плоская - это типа была аксиома!!! Но оказалось неправдой...

nikitos
24.10.2010, 23:05
Тема ни о чем! Простой обыватель, пытается выяснить, возможно ли обнаружить центр вселенной - то место где как утверждается произошел так называемый БОЛЬШОЙ ВЗРЫВ - место где находилось то микроскопическое нечто, которое начало расширяться. И что мы имеем в итоге - умные дяденьки говорят, что обыватель дурак - и этого места нет... что ж, спасибо. Мы живем не глубокой старине, и знаем, то земля не центр мира и вселенной. Но кажется, что умные дяденьки должны нам, глупым обьяснить, как понимать, что вселенная - это сфера или то, что она плоская? какова толщина этой самой сферы? налево и направо по 13,7 млр св.лет, а вверх и вниз сколько, а под углом 45 град?Помнится когда то утверждалось, что земля плоская - это типа была аксиома!!! Но оказалось неправдой...
Сюда постучитесь: http://www.astronomy.ru/forum/index.php/topic,79268.0.html?PHPSESSID=rugcblo4s3hdho8o11tkf 7src0

thirtyseconds
25.10.2010, 01:09
как понимать, что вселенная - это сфера или то, что она плоская? какова толщина этой самой сферы? налево и направо по 13,7 млр св.лет

Это не физическое описание, а аналогия, которая используется для того что бы было проще понять как может что-то расширяться и при этом не иметь центра расширения.

Хотите физического описания, читайте учебники по космологии. Умные дяденьки их для того и пишут.

Roman2008
25.10.2010, 22:06
А если подойти с другой стороны: вот Земля вращается вокруг Солнца, Солнце вокруг центра галактики, галактика вращается вокруг... закончите движение дальше.

thirtyseconds
26.10.2010, 00:54
Можно подойти, а можно не подходить... Совсем не факт, что если что-то вращается тут, то должно вращаться всё и везде.

Барсик
26.10.2010, 10:00
Можно подойти, а можно не подходить... Совсем не факт, что если что-то вращается тут, то должно вращаться всё и везде.

Однако известно, что галактики располагаются группами. Возможно и вращаются, вот только периоды получаются очень большие:D

igor_da_bari
26.10.2010, 10:29
Удивительно глубокое замечание :)

Было бы смертельно удивительно, противоречило бы всему, что мы знаем о гравитации и составило бы величайшую загадку, если бы галактики в скоплениях не двигались относительно друг друга или двигались так, что в их взаимном движении не было тангенциальной составляющей...

Таки да, про вращение галактик в скоплениях говорят даже продавцы фруктов на Лефортовском рынке. Они таки вращаются. Без всяких "возможно".

Барсик
26.10.2010, 10:44
Таки да, про вращение галактик в скоплениях говорят даже продавцы фруктов на Лефортовском рынке.

Интеллигентные продавцы, однако, на Лефортовском рынке. Какие темы поднимают:eek:. Не иначе как тюрьма на них так влияет:D


Они таки вращаются. Без всяких "возможно".

И вокруг чего же они вращаются?

igor_da_bari
26.10.2010, 11:00
Да, народ нынче хоть и с грехом пополам, но среднее образование в целом освоил и знает ответы хотя бы на самые элементарные вопросы. Исключение составляют отдельные читатели и писатели этой темы :D :D :D

А что касается того, вокруг чего вращаются Луна, Земля, Солнце, Галактика и тыды - вы уж будьте добры, школьные учебники полистайте. Или там "Справочник астронома-любителя" Куликовского. Ну, если вам это интересно, конечно. А если нет, то листать не нужно, но тогда не стоит и просить людей писать специально для вас эту литературу еще раз :p

Я понимаю, что некоторым проще кликнуть клич на форуме: "Какое седня число?!", чем заглянуть в календарь. Но мне лично такой способ получения информации не нравится. Паразитический он какой-то...

Roman2008
26.10.2010, 21:13
А что касается того, вокруг чего вращаются Луна, Земля, Солнце, Галактика и тыды....
Да я не спрашиваю, предпологаю, что во Вселенной всё движется и движется бесконечно. И как такового центра, я считаю, нет.
Или есть такое тело, которое не вращается вокруг чего-то.

igor_da_bari
26.10.2010, 21:59
Да я не спрашиваю, предпологаю, что во Вселенной всё движется и движется бесконечно. И как такового центра, я считаю, нет.
Или есть такое тело, которое не вращается вокруг чего-то.

Во Вселенной все и точно движется. Это давно подмечено. А бесконечно или нет - это смотря по тому, что вы понимаете под бесконечностью.

Центра до сих пор никто не обнаружил (совершенно, нужно сказать, независимо от того, что считаете лично вы и на каком основании вы это считаете).

Любое тело в системе отсчета связанной с ним, не вращается вокруг ничего. Так что таких тел хоть завались. Любое тело по выбору может не вращаться вокруг чего-то.

В общем выходит, что вы кругом правы. Вопрос только, если от этого хоть кому-то хоть какая-то польза...

Gennady Bankewitch
26.10.2010, 23:22
Вот хрен с ним, с центром. Всё никак не могу вообразить, ну почему тёмную материю помещают куда угодно, но не в Солнечную систему? Ну, понимаю, распределена не равномерно, больше достаётся галактикам, центрам галактик, и, вообще, пофиг как. В принципе понимаю зачем придумана. Но почему влияние ТМ столь невзрачно в пределах родного нам пространства, нет её тут что-ли?

vvv2007
27.10.2010, 00:40
Да и с темной материей тоже хрен. Есть еще темная энергия. Вот она вообще... Мало того, что она темная, так она еще и Вселенную расширяет!

КентаVR
27.10.2010, 08:12
Да и с темной материей тоже хрен. Есть еще темная энергия
Которые связаны между собой через скорость тьмы в квадрате :)

igor_da_bari
27.10.2010, 10:34
Вот хрен с ним, с центром. Всё никак не могу вообразить, ну почему тёмную материю помещают куда угодно, но не в Солнечную систему? Ну, понимаю, распределена не равномерно, больше достаётся галактикам, центрам галактик, и, вообще, пофиг как. В принципе понимаю зачем придумана. Но почему влияние ТМ столь невзрачно в пределах родного нам пространства, нет её тут что-ли?

Просто потому что плотность у нее маленькая и на маленьких масштабах "родного пространства" она практически никак не проявляется. Иначе ее давно бы увидели на этих масштабах. А так - заметили только на галактических. Так уж природа устроила, ничего не попишешь.

conus
11.11.2011, 01:59
Так уж природа устроила, ничего не попишешь.
Зачем тогда использовать термин Большой Взрыв?.. - ведь взрыв имеет эпицентр. И что такое природа, Вы о чем?

thirtyseconds
11.11.2011, 12:12
В английском его и не используют.

conus
11.11.2011, 14:19
Т.е, в сегодняшнем эпицентре передовой научной мысли, на БАКе, про Большой Взрыв говорят не на английском?

thirtyseconds
11.11.2011, 20:27
У вас к чему претензия ? Назвать можно и "гигантским бутербродом" и говорить на португальском. Можно в БАКе, можно в коробке.
Само явление описывается не словами, а формулами.

Грин
11.11.2011, 20:36
Зачем тогда использовать термин Большой Взрыв?.. - ведь взрыв имеет эпицентр.?
Воспользуйтесь поиском - Объёмный взрыв. :D

И что такое природа, Вы о чем?
Не приставайте к человеку - он давно уже не может вам ответить...:(

conus
11.11.2011, 22:03
У вас к чему претензия ?
Что-то непонятно из вступительного поста?..

Назвать можно и "гигантским бутербродом"И большим презервативом, судя по всему, вы бы назвали, но назвали Большим Взрывом, простите, что забыли с вами посоветоваться.

и говорить на португальском. Можно в БАКе, можно в коробке.Лишь бы не на том, который вы избрали для общения и не оттуда, откуда вы вещаете.

Само явление описывается не словами, а формулами.Когда будет у этого явления ВАШЕ название и я его исковеркаю, тогда начинайте за.упаться. А пока ни название не ваше, ни формулы, ни понимания примитивного закономерного вопроса.

Воспользуйтесь поиском - Объёмный взрыв. :D

Зачем поиск? ЛЮБОЙ взрыв зарождается из определенного объема в пространстве. Однако, будьте любезны, покажите, пожалуйста, где говориться, что у объемного взрыва нет центра?

Ar-Gen-Tum
11.11.2011, 22:15
Что-то непонятно из вступительного поста?..

И большим презервативом, судя по всему, вы бы назвали, но назвали Большим Взрывом, простите, что забыли с вами посоветоваться.

...

1. "Big Bang theory". Типа (по русски) - "Большой Бумк". :D
А взрыв по ихнему - explode (эксполозия). При взрыве что-то начинает расширяться от центра.
2. Не торопитесь верить в то что пишут. Хотя ... - это Ваше личное дело.
(Если Вы прочитали про "Большой Бумк" и поверили в него, это еще
вовсе не означает, что все именно так и было. ;) )

conus
11.11.2011, 23:12
Ar-Gen-Tum (http://starlab.ru/member.php?u=15938)
Хорошо, пусть англичане предусмотрительно бумкают, но мы-то взрываем :). Какую терминологию МЫ используем, ту и пользую. А что есть в их понимании этот бумк, как не распространение материи из объема пространства? - те же признаки взрыва, так что эпицентр никак не обойти.

Не торопитесь верить в то что пишут. Хотя ... - это Ваше личное дело.
(Если Вы прочитали про "Большой Бумк" и поверили в него, это еще
вовсе не означает, что все именно так и было. ;) )Пока ниоткуда не следует, что я в него верю :), просто захотелось уточнить очевидные, на мой взгляд, вещи.

Ar-Gen-Tum
11.11.2011, 23:25
[URL="http://starlab.ru/member.php?u=15938"]
...
А что есть в их понимании этот бумк, как не распространение материи из объема пространства? - те же признаки взрыва, так что эпицентр никак не обойти.
"Вопрос конечно интересный."
Когда-то, довольно давно, я это понимал, ... ибо не задумывался особо. :)
...

conus
11.11.2011, 23:40
Дык я раньше тоже крепко спал, оттого, что меньше понимал. Сейчас с этим вопросом не могу сказать, что засыпаю, но, то, что "вопрос интересный" - ничего не сказать. Хотя, поголовное большинство математиков и физиков склонно замыливать рассуждения по этому поводу, выставляя аналогом явлений формулы с бесконечностями. Для оценки структуры Мироздания требуется только логика, я надеюсь, мы не вываливаемся из темы? - ведь явление центра вселенной претендует на попытку объяснить происходящее.

Ar-Gen-Tum
12.11.2011, 00:06
...
Сейчас с этим вопросом не могу сказать, что засыпаю, но, то, что "вопрос интересный" - ничего не сказать.
...
История вопроса прибизительно такова (ИМХО):
1. Исследователи мироздания полагают, что Вселенная везде примерно одинакова.
Полагать иное нет вроде-бы оснований.
2. При исследовании глубокого космоса, в удаленных объектах наблюдается
смещение спектральных линий. Исходя из п.1 их отождествляют с эффектом Доплера-Физо.
3. Самые удаленные наблюдаемые объекты - квазары.
Вычисленные расстояния до них примерно одинаковы, +- пару миллиард световых лет.
Получается, что видимая Вселенная расширяется. А центр расширения
находится на Земле. :)
Но это мракобесие, считать Землю центром Вселенной. :)
Посему придуманы витиеватые объяснения.
И хотя возраст Вселенной вычислен, исходя из наблюдений,
из вышеизложенного следует, что наблюдатель из системы далекого квазара
должен видеть примерно такую-же картину, как и наблюдатель с Земли.
Т.е. самый удаленный квазар с его позиции будет так-же удален от него,
как и он удален от наблюдателя с Земли.
...